download scanned papers here - Paper 1    Paper II


UPSC PRELIMS 2022 - PAPER I - ANSWER KEY

Exam Analysis HOME       2022 - Paper I - English       Paper II - E       Paper I - Hindi       Paper II - H      



BIRD'S EYEVIEW

Exam Analysis HOME       2022 - Paper I - English       Paper II - E       Paper I - Hindi       Paper II - H      

  • संघ लोक सेवा आयोग (UPSC) प्रारंभिक 2022 पेपर 1 और 2 का आयोजन आयोग द्वारा देश भर के विभिन्न केंद्रों पर किया गया था।
  • परीक्षा तिथि: 5 जून, 2022 (रविवार) | समय: 09:30 पूर्वाह्न से 11:30 पूर्वाह्न (दो घंटे) | परीक्षा की प्रकृति: बहुविकल्पीय परीक्षण पेन और पेपर टेस्ट | प्रश्नों की कुल संख्या : 100 | कुल अंक: प्रत्येक गलत उत्तर के लिए 33% नकारात्मक अंकन के साथ 200 अंक का पेपर।
  • नए तरह के प्रश्न: इस वर्ष सामान्य अध्ययन पेपर में प्रश्नों की एक नई किस्म पेश की गई, जहां प्रश्न ने उम्मीदवारों से मिलान विकल्पों की सही संख्या चुनने के लिए कहा। इस पैटन पर कुछ प्रश्न "अंतर्राष्ट्रीय संबंध" पूछे गए थे। नया प्रश्न पैटर्न कठिन और समय लेने वाला था। इस प्रकार के कुल 7 प्रश्न थे। इसका असर कट-ऑफ पर पड़ेगा।
  • वापसी : मध्यकालीन इतिहास की शब्दावली और अंतर्राष्ट्रीय संबंधों पर प्रश्न पुनः पूछे गए। अंतरराष्ट्रीय क्षेत्रों, संगठनों और सम्मेलनों पर अधिक संख्या में प्रश्न पूछे गए थे। अफगानिस्तान, मध्य एशिया, पूर्वी यूरोप और अफ्रीका के बारे में प्रश्न पूछे गए थे।
  • करेंट अफेयर्स का लिंकेज: प्रश्नों को करेंट अफेयर्स और स्थिर ज्ञान के बीच ज्ञान को जोड़ने की आवश्यकता थी। लेकिन इन्हे हल करना ज्यादा मुश्किल नहीं था।
  • संतुलित कठिनाई स्तर: इस वर्ष के प्रश्न पत्र में यूपीएससी पाठ्यक्रम में उल्लिखित सभी वर्गों को अच्छे वेटेज के साथ कवर किया गया है।टेस्ट आम तौर पर एक-समान था। सभी प्रमुख विषयों को लगभग समान वेटेज दिया गया है। संतुलित वितरण। लेकिन 2023 में क्या होगा इसकी कोई गारंटी नहीं है!
  • कठोरता: पहली नज़र में, पेपर सामान्य लगता है क्योंकि विषय परिचित लगते हैं लेकिन शैतानी विवरण में निहित है! सतही ज्ञान पर्याप्त नहीं था; पेचीदगियों ने इस बार फर्क किया।
  • वैचारिक स्पष्टता: यह सर्वोपरि थी। कई प्रश्न कठिन थे - जहां अवधारणाओं की स्पष्टता सर्वोपरि थी। एक अवधारणा की परिभाषा जानना पर्याप्त नहीं था, विशेष रूप से अर्थव्यवस्था में कई उपयोगिता-आधारित प्रश्न थे।
  • मानचित्रण: इन प्रश्नो को, इस बार (पिछले वर्षों की तुलना में) प्रश्नों का अधिक हिस्सा मिला। अधिकांश समाचारों में थे- अत: एटलस के साथ समाचार पत्र पढ़ना अब साधना करने की एक अच्छी आदत है। घरेलू (आर्द्रभूमि, नदियाँ) और अंतर्राष्ट्रीय क्षेत्र (लेवांत, अफगानिस्तान सीमा, आदि ) समान रूप से महत्वपूर्ण थे।
  • विषयवार विश्लेषण:
    1. पर्यावरण और पारिस्थितिकी में कुछ अच्छे प्रश्न देखे गए, जिनमें कई सूक्ष्म विवरण पूछे गए। कुल 15 प्रश्न उपस्थित हुए (पिछले वर्ष 21)
    2. विज्ञान और प्रौद्योगिकी के प्रश्न संख्या में 12 (पिछले वर्ष 4) थे। आईटी और संचार प्रौद्योगिकी पर ध्यान केंद्रित किया गया था
    3. राजनीति के प्रश्न तथ्यात्मक थे लेकिन संभव थे और पिछले वर्षों की तुलना में अपेक्षाकृत आसान थे। कुल 10 पूछे गए थे (पिछले साल 17)
    4. अर्थव्यवस्था और बैंकिंग प्रश्न समसामयिक घटनाओं से संबंधित थे और अनुमान लगाने के लिए भी अवधारणाओं की गहरी समझ की मांग करते थे। सतही ज्ञान मदद नहीं करेगा।
    5. प्राचीन और मध्यकालीन इतिहास, और कला और संस्कृति के प्रश्न कठिन थे - विवरण और शब्दावली के बहुत सारे याद रखने की आवश्यकता थी। फोकस क्षेत्र बौद्ध धर्म, जैन धर्म, मंदिर, साहित्य और ग्रंथ, मंगोल, प्रारंभिक ब्रिटिश विस्तार, संवैधानिक इतिहास, क्रांतिकारी, संगम साहित्य, मौर्य राजवंश थे। कुछ प्रश्नों का उत्तर देना कठिन कहा जा सकता है।
    6. दो प्रश्न सरकारी योजनाओ पर पूछे गए।
    7. भूगोल के प्रश्न कम वैचारिक थे, और तथ्यात्मक और स्थान-आधारित प्रश्नों का मिश्रण पूछा गया था।
  • रणनीति: पेपर चतुराई से तैयार किया गया था: कुछ प्रश्नों में उन्मूलन रणनीतियों के उपयोग को रोक दिया गया था, और सभी उत्तरों के बारे में वास्तविक ज्ञान की आवश्यकता थी। दिलचस्प बात यह है कि विकल्पों के एक नए पैटर्न के कारण उन्मूलन तकनीकों का उपयोग करने का दायरा हटा दिया गया था। एक आकांक्षी को लगातार प्रयास करने होते हैं और कई संशोधन करने पड़ते हैं।
  • अपेक्षित कट-ऑफ: प्रश्न पत्र के स्तर को ध्यान में रखते हुए, कट-ऑफ 90-95 की सीमा में होने का अनुमान है; पिछले साल की कट ऑफ (सामान्य श्रेणी के लिए 87.54) से अधिक है।
TOPICWISE DISTRIBUTION

Exam Analysis HOME       2022 - Paper I - English       Paper II - E       Paper I - Hindi       Paper II - H      



DETAILED QUESTIONWISE SOLUTIONS (Set B)

Exam Analysis HOME       2022 - Paper I - English      Paper II - E       Paper I - Hindi       Paper II - H      


1. “जलवायु कार्रवाई ट्रैकर (क्लाइमेट ऐक्शन ट्रैकर)” जो विभिन्न देशों के उत्सर्जन अपचयन के लिए दिए गए वचनों की निगरानी करता है, क्या है?

  1. अनुसंधान संगठनों के गठबंधन द्वारा निर्मित डेटाबेस
  2. “जलवायु परिवर्तन के अंतर्राष्ट्रीय पैनल" का स्कंध (विंग)
  3. “जलवायु परिवर्तन पर संयुक्त राष्ट्र ढाँचा अभिसमय” के अधीन समिति
  4. संयुक्त राष्ट्र पर्यावरण कार्यक्रम और विश्व बैंक द्वारा संवर्धित और वित्तपोषित एजेंसी

Sol. Ans.(a). क्लाइमेट एक्शन ट्रैकर एक स्वतंत्र वैज्ञानिक विश्लेषण है जो सरकार की जलवायु कार्रवाई को ट्रैक करता है और इसे विश्व स्तर पर सहमत पेरिस समझौते जिसका उद्देश्य "वार्मिंग को 2 डिग्री सेल्सियस से नीचे अच्छी तरह से रोकना और वार्मिंग को 1.5 डिग्री सेल्सियस तक सीमित करने के प्रयास करना है", के समक्ष मापता है। यह दो संगठनों - क्लाइमेट एनालिटिक्स और न्यू क्लाइमेट इंस्टीट्यूट का सहयोग है। यह 2009 से स्वतंत्र विश्लेषण प्रदान कर रहा है। यह आईपीसीसी, यूएनएफसीसी, यूएनईपी और डब्ल्यूबी (विश्व बैंक) से संबंधित नहीं है। अतः, विकल्प (b), (c) और (d) सत्य नहीं हैं।

( यह प्रश्न सीधे पीटी के पावरपैक कोर्स में शामिल था - यहां नामांकन करें -https://bit.ly/upscias )

स्रोत: https://climateactiontracker.org/about/


2. निम्नलिखित कथनों पर विचार कीजिए:

  1. “जलवायु समूह (दि क्लाइमेट ग्रुप)” एक अंतर्राष्ट्रीय गैर-लाभकारी संगठन है जो बड़े नेटवर्क. बना कर जलवायु क्रिया को प्रेरित करता है और उन्हें चलाता है।
  2. अंतर्राष्ट्रीय ऊर्जा एजेंसी ने जलवायु समूह की भागीदारी में एक वैश्विक पहल “EP100" प्रारंभ की।
  3. EP100, ऊर्जा दक्षता में नवप्रवर्तन को प्रेरित करने एवं उत्सर्जन न्यूनीकरण लक्ष्यों को प्राप्त करते हए प्रतिस्पर्धात्मकता बढ़ाने के लिए प्रतिबद्ध अग्रणी कंपनियों को साथ लाता है।
  4. कुछ भारतीय कंपनियाँ EP100 की सदस्य हैं।
  5. अंतर्राष्ट्रीय ऊर्जा एजेंसी “अंडर 2 कोएलिशन” का सचिवालय है। उपर्युक्त कथनों में कौन-से सही हैं?

उपर्युक्त कथनों में से कौन - से सही हैं ?

  1. 1, 2, 4 और 5
  2. केवल 1, 3 और 4
  3. केवल 2, 3 और 5
  4. 1, 2, 3, 4 और 5

Sol. Ans.(b). "जलवायु समूह" अंडर 2 कोएलिशन का सचिवालय है, न कि अंतर्राष्ट्रीय ऊर्जा एजेंसी का जैसा कि कथन 5 में दिया गया है। इसलिए कथन 5 सही उत्तर का हिस्सा नहीं होना चाहिए। ऐसा विकल्प केवल (b) है।

स्रोत:
https://www.theclimategroup.org/frequently-asked-questions#:~:text=The%20Climate%20Group%20is%20the,holding%20the%20governments%20to%20account.


3. “यदि वर्षावन और उष्णकटिबंधीय वन पृथ्वी के फेफड़े हैं, तो निश्चित ही आर्द्रभूमियाँ इसके गुर्दो की तरह काम करती हैं ।” निम्नलिखित में से आर्द्रभूमियों का कौन-सा एक कार्य उपर्युक्त कथन को सर्वोत्तम रूप से प्रतिबिंबित करता है?

  1. आर्द्रभूमियों के जल चक्र में सतही अपवाह, अवमृदा अंत:स्रवण और वाष्पन शामिल होते हैं ।
  2. शैवालों से वह पोषक आधार बनता है, जिस पर मत्स्य, परुषकवची (क्रश्टेशिआई), मृदुकवची (मोलस्क), पक्षी, सरीसृप और स्तनधारी फलते-फूलते हैं।
  3. आर्द्रभूमियाँ अवसाद संतुलन और मृदा स्थिरीकरण बनाए रखने में महत्त्वपूर्ण भूमिका निभाती हैं।
  4. जलीय पादप भारी धातुओं और पोषकों के आधिक्य को अवशोषित कर लेते हैं ।

Sol. Ans.(d). जबकि उष्णकटिबंधीय वर्षावनों को अक्सर पृथ्वी के फेफड़े कहा जाता है, आर्द्रभूमि को गुर्दे के रूप में वर्णित किया जाता है। ऐसा इसलिए कहा जाता है क्योंकि वे बड़ी मात्रा में पानी को अवशोषित कर सकते हैं, प्रदूषकों को छान सकते हैं और बाढ़ को रोकने में मदद कर सकते हैं। जलीय पौधे जलीय पारिस्थितिक तंत्र में भारी धातुओं के लिए सिंक होते हैं , और विभिन्न जलीय पौधों की प्रजातियां विभिन्न भारी धातुओं की परिवर्तनीय मात्रा जमा करती हैं । उदाहरण के लिए, अध्ययनों से पता चला है कि तैरते हुए पौधे जैसे जल जलकुंभी (ईचोर्निया क्रैसिप्स), इमर्जेंट पौधे जैसे कैटेल (टाइफा। लैटिफोलिया), और जलमग्न पौधे जैसे कि हाइड्रिला वर्टिसिलाटा, सेराटोफिलम डेमर्सम, और पोटामोगेटन मैलियनस में भारी धातुओं को जमा करने की महत्वपूर्ण क्षमता होती है। वे जलवायु परिवर्तन के खिलाफ लड़ाई में प्रमुख सहयोगी भी बन गए हैं, क्योंकि वे वातावरण से CO2 को अवशोषित कर सकते हैं। वनों की तुलना में आर्द्रभूमि अभी भी तीन गुना तेजी से गायब हो रही है। (विकल्प (c) भी काफी करीब है)

( यह प्रश्न सीधे पीटी के पावरपैक कोर्स में शामिल था - यहां नामांकन करें -https://bit.ly/upscias )

स्रोत: https://www.ncbi.nlm.nih.gov/pmc/articles/PMC6313464/


4. WHO के वायु गुणवत्ता दिशानिर्देशों के सन्दर्भ में, निम्नलिखित कथनों पर विचार कीजिए:

अन्य बातें अपरिवर्तित रहने पर भी किसी वस्तु के लिए बाजार मांग बढ़ सकती है, यदि

  1. PM2.5 का 24-घंटा माध्य 15 µg/m3 से अधिक नहीं बढ़ना चाहिए और PM का वार्षिक माध्य 5µg/m3 से अधिक नहीं बढ़ना चाहिए ।
  2. किसी वर्ष में, ओज़ोन प्रदूषण के उच्चतम स्तर प्रतिकूल मौसम के दौरान होते हैं ।
  3. PM10 फेफड़े के अवरोध का वेधन कर रक्त-प्रवाह में प्रवेश कर सकता है।
  4. वायु में अत्यधिक ओज़ोन दमा को उत्पन्न कर ___ सकती है। उपर्युक्त कथनों में कौन-से सही हैं?

उपर्युक्त कथनों में से कौन-से सही है?

  1. 1, 3 और 4
  2. केवल 1 और 4
  3. 2, 3 और 4
  4. केवल 1 और 2

Sol. Ans.(b). कथन 3 - पीएम 2.5 फेफड़े में प्रवेश कर सकता है और रक्तप्रवाह में आ सकता है लेकिन पीएम 10 नहीं। इसलिए, 3 सही उत्तर विकल्प का हिस्सा नहीं होना चाहिए। इसलिए (a) और (c) को खारिज किया जा सकता है। 2 और 4 में से, यह सच है कि अत्यधिक ओजोन अस्थमा शरीर में ला सकता है, जबकि प्रतिकूल मौसम के दौरान ओजोन प्रदूषण के उच्चतम स्तर के प्रमाण स्पष्ट नहीं हैं। कथन 1 सही है (कृपया नीचे दिए गए स्रोत को देखें)।

( यह प्रश्न सीधे पीटी के पावरपैक कोर्स में शामिल था - यहां नामांकन करें -https://bit.ly/upscias )

स्रोत 1: https://www.breeze-technologies.de/blog/new-2021-who-air-quality-guideline-limits/

स्रोत 2:
https://www.epa.gov/ground-level-ozone-pollution/health-effects-ozone-pollution#:~:text=Long%2Dterm%20exposure%20to%20ozone,with%20deaths%20from%20respiratory%20causes.


5. कभी-कभी समाचारों में उल्लिखित “गुच्छी”के सन्दर्भ में, निम्नलिखित कथनों पर विचार कीजिए:

  1. यह एक कवक है।
  2. यह कुछ हिमालयी वन क्षेत्रों में उगती है।
  3. उत्तर-पूर्वी भारत के हिमालय की तलहटी में इसकी वाणिज्यिक रूप से खेती की जाती है । उपर्युक्त कथनों में कौन-सा/से सही है/हैं?
  1. केवल 1
  2. केवल 3
  3. 1 और 2
  4. 2 और 3

Sol. Ans.(c). गुच्छी सबसे महंगे मशरूम में से एक के रूप में जाना जाता है, जिसमें से केवल 500 ग्राम की कीमत 18000 रुपये तक होती है। मशरूम क्या है? मशरूम या टॉडस्टूल एक कवक का मांसल, बीजाणु भाग है, जो आमतौर पर जमीन के ऊपर, मिट्टी पर या उसके खाद्य स्रोत पर उत्पन्न होता है। अतः 1 सही है और विकल्प (b) और (c) को खारिज किया जा सकता है। 2 भी सही है क्योंकि हिमपात के बाद हिमालयी क्षेत्र में गुच्छी मशरूम उगाए जाते हैं • उनके पास एक अद्वितीय स्वाद और मिट्टी की सुगंध होती है (अधिक जानकारी के लिए स्रोत देखें)।

स्रोत:
https://economictimes.indiatimes.com/gucchi-wild-mushrooms-from-himalayas-worth-their-weight-in-gold/articleshow/19007096.cms?from=mdr


6. पॉलीएथिलीन टेरेपथलेट के सन्दर्भ में, जिसका हमारे दैनिक जीवन में बहुत व्यापक उपयोग है, निम्नलिखित कथनों पर विचार कीजिए:

  1. इसके तंतुओं को ऊन और कपास के तंतुओं के साथ, उनके गुणधर्मों को प्रबलित करने हेतु, सम्मिश्रित किया जा सकता है।
  2. इससे बने पात्रों को किसी भी मादक पेय को रखने के लिए उपयोग किया जा सकता है।
  3. इससे बनी बोतलों का पुनर्चक्रण (रीसाइक्लिंग) कर उनसे अन्य उत्पाद बनाए जा सकते हैं ।
  4. इससे बनी वस्तुओं का भस्मीकरण द्वारा, बिना ग्रीनहाउस गैस उत्सर्जन किए, आसानी से निपटान किया जा सकता है।

उपर्युक्त कथनों में कौन-से सही हैं?

  1. 1 और 3
  2. 2 और 4
  3. 1 और 4
  4. 2 और 3

Sol. Ans.(a). पीईटी (या पीईटीई) एक थर्मोप्लास्टिक है, थर्मोसेट प्लास्टिक नहीं। अतः इसे भस्म नहीं किया जा सकता बल्कि इसके बजाय तरल में बदल जाएगा। अतः कथन 4 गलत है, और (b) और (c) को खारिज किया जा सकता है। इसके अलावा, पीईटी को कुछ फर्मों द्वारा विपणन किए जाने वाले उच्च शक्ति वाले कपड़ा फाइबर में बनाया जाता है। पीईटी फाइबर की कठोरता उन्हें विरूपण के लिए अत्यधिक प्रतिरोधी बनाती है, इसलिए वे कपड़ों में झुर्रियों के लिए उत्कृष्ट प्रतिरोध प्रदान करते हैं। वे अक्सर रेयान, ऊन और कपास जैसे अन्य फाइबर के साथ टिकाऊ-प्रेस मिश्रणों में उपयोग किए जाते हैं, जो उन तंतुओं के अंतर्निहित गुणों को मजबूत करते हैं, जबकि कपड़े की झुर्रियों से उबरने की क्षमता में योगदान करते हैं। अतः 1 सही है, विकल्प (b) और (d) को खारिज कर दिया गया है। पीईटी सबसे व्यापक रूप से पुनर्नवीनीकरण प्लास्टिक है। पीईटी बोतलों और कंटेनरों को आमतौर पर पिघलाया जाता है और फाइबरफिल या कालीन के लिए फाइबर में काता जाता है। अतः 3 सही है।

स्रोत 1: https://www.britannica.com/science/polyethylene-terephthalate#ref1049366

स्रोत 2 : https://www.thehindu.com/news/cities/mumbai/news/Govt-rules-out-plastic-liquor-bottles/article13994794.ece


7. निम्नलिखित में से कौन-सा पक्षी नहीं है?

  1. गोल्डन महासीर
  2. इंडियन नाइटजार
  3. स्पूनबिल
  4. व्हाईट आइबिस

Sol. Ans.(a). आसान प्रश्न। गोल्डन महासीर एक मछली है, जिसे " संकटग्रस्त" कहा जाता है। अन्य तीन पक्षी प्रजातियां हैं।

स्रोत: https://www.wwfindia.org/about_wwf/priority_species/threatened_species/golden_mahseer/


8. निम्नलिखित में कौन-से, नाइट्रोजन-यौगिकीकरण पादप हैं?

  1. अल्फाल्फा
  2. चौलाई (ऐमरंथ)
  3. चना (चिक-पी)
  4. तिपतिया घास (क्लोवर)
  5. कुलफा (पर्सलेन)
  6. पालक

नीचे दिए कूट का प्रयोग कर सही उत्तर चुनिए:

  1. केवल 1, 3 और 4
  2. केवल 1, 3, 5 और 6
  3. केवल 2, 4, 5 और 6
  4. 1, 2, 4, 5 और 6

Sol. Ans.(a). यह कृषि से एक प्रश्न है। नाइट्रोजन स्थिरीकरण में योगदान देने वाले पौधों में फलियां परिवार - फैबेसी - शामिल हैं, जैसे कि तिपतिया घास, सोयाबीन, अल्फाल्फा, ल्यूपिन, मूंगफली, और रूइबोस। उनके रूट सिस्टम में नोड्यूल्स के भीतर राइज़ोबिया नामक सहजीवी बैक्टीरिया होते हैं, जो नाइट्रोजन यौगिकों का उत्पादन करते हैं जो पौधे को बढ़ने और अन्य पौधों के साथ प्रतिस्पर्धा करने में मदद करते हैं। जब पौधा मर जाता है, अतः निश्चित नाइट्रोजन निकलता है, जिससे यह अन्य पौधों को उपलब्ध होता है और इससे मिट्टी को उर्वरित करने में मदद मिलती है। अधिकांश फलियों में यह जुड़ाव होता है, लेकिन कुछ जेनेरा (जैसे, स्टाईफ्नोलोबियम) नहीं होते हैं। कई पारंपरिक और जैविक कृषि पद्धतियों में, खेतों को विभिन्न प्रकार की फसलों के माध्यम से घुमाया जाता है, जिसमें आमतौर पर मुख्य रूप से या पूरी तरह से तिपतिया घास या एक प्रकार का अनाज (पारिवारिक पॉलीगोनेसी) शामिल होता है, जिसे अक्सर "हरी खाद" कहा जाता है।

स्रोत:
https://en.wikipedia.org/wiki/Category:Nitrogen-fixing_crops#:~:text=Plants%20that%20contribute%20to%20nitrogen,lupins%2C%20peanuts%2C%20and%20rooibos.


9. निम्नलिखित स्थितियों में से किस एक में “जैवशैल प्रौद्योगिकी (बायोरॉक टेक्नोलॉजी)” की बातें होती हैं?

  1. क्षतिग्रस्त प्रवाल भित्तियों (कोरल रीफ्स) की बहाली
  2. पादप अवशिष्टों का प्रयोग कर भवन-निर्माण सामग्री का विकास
  3. शेल गैस के अन्वेषण/निष्कर्षण के लिए क्षेत्रों की पहचान करना
  4. वनों/संरक्षित क्षेत्रों में जंगली पशुओं के लिए लवण-लेहिकाएँ (साल्ट लिक्स) उपलब्ध कराना

Sol. Ans.(a). "बायोरॉक" तकनीक एक अभिनव प्रक्रिया है जिसका मूल रूप से 1976 में वुल्फ हिल्बर्ट्ज़ द्वारा समुद्र में प्राकृतिक निर्माण सामग्री का उत्पादन करने के लिए आविष्कार किया गया था। बायोरॉक सामग्री एकमात्र समुद्री निर्माण सामग्री है जो बढ़ती है, उम्र के साथ मजबूत होती है, और स्वयं की मरम्मत करती है। प्रौद्योगिकी को बायोरॉक कहा जाता था, क्योंकि इसने न केवल संरचनात्मक उद्देश्यों के लिए कठोर चूना पत्थर की चट्टान को विकसित किया, बल्कि इसने कोरल और सभी समुद्री जीवों की वृद्धि में काफी वृद्धि की। वैश्विक ऊष्मीकरण के कारण प्रवाल भित्तियाँ खतरे में हैं, और इसलिए इस तकनीक को बहुत उपयोगी माना जाता है।

( यह प्रश्न सीधे पीटी के पावरपैक कोर्स में शामिल था - यहां नामांकन करें -https://bit.ly/upscias )

Source : https://www.globalcoral.org/biorock-coral-reef-marine-habitat-restoration/


10. “मियावाकी पद्धति”किसके लिए विख्यात है?

  1. शुष्क और अर्ध-शुष्क क्षेत्रों में वाणिज्यिक कृषि का संवर्धन
  2. आनुवंशिकत: रूपांतरित पुष्पों का प्रयोग कर उद्यानों का विकास
  3. शहरी क्षेत्रों में लघु वनों का सृजन
  4. तटीय क्षेत्रों और समुद्री सतहों पर पवन ऊर्जा का संग्रहण

Sol. Ans.(c). "मियावाकी" जापानी वनस्पतिशास्त्री अकीरा मियावाकी द्वारा दी गई तकनीक है, जो घने, देशी जंगलों के निर्माण में मदद करती है, और शहरी गर्म द्वीपों के सबसे बुरे प्रभावों को कम करने में बहुत उपयोगी हो सकती है। दृष्टिकोण यह सुनिश्चित करने के लिए माना जाता है कि पौधों की वृद्धि 10 गुना तेज हो और परिणामी वृक्षारोपण सामान्य से 30 गुना अधिक सघन हो। अतः, (c) सही उत्तर है।

( यह प्रश्न सीधे पीटी के पावरपैक कोर्स में शामिल था - यहां नामांकन करें -https://bit.ly/upscias )

स्रोत:
https://www.sei.org/about-sei/press-room/how-the-miyawaki-method-can-transform-indian-cities/#:~:text=The%20method%20involves%20planting%20two,insects%2C%20and%20create%20carbon%20sinks.


11. निम्नलिखित पर विचार कीजिए:

  1. आरोग्य सेतु
  2. कोविन
  3. डिजीलॉकर
  4. दीक्षा

उपर्युक्त में से कौन-से, ओपेन-सोर्स डिजिटल प्लेटफॉर्म पर बनाए गए हैं?

  1. केवल 1 और 2
  2. केवल 2, 3 और 4
  3. केवल 1, 3 और 4
  4. 1, 2, 3 और 4

Sol. Ans.(d). अन्य सभी ऐप्स कोविन, डिजीलॉकर और दीक्षा ओपन-सोर्स डिजिटल प्लेटफॉर्म पर बनाए गए हैं। इसे FOSS - फ्री और ओपन सोर्स सॉफ्टवेयर कहा जाता है, जिसे भारत सरकार ने कई सार्वजनिक रूप से उपयोग किए जाने वाले अनुप्रयोगों के लिए उपयोग किया है। आरोग्य सेतु मूल रूप से उस तरह से नहीं बनाया गया था। यह एक भारतीय कोविड-19 "संपर्क अनुरेखण, सिंड्रोमिक मैपिंग और स्व-मूल्यांकन" डिजिटल सेवा है, जो मुख्य रूप से एक मोबाइल ऐप है, जिसे इलेक्ट्रॉनिक्स और सूचना प्रौद्योगिकी मंत्रालय के तहत राष्ट्रीय सूचना विज्ञान केंद्र द्वारा विकसित किया गया है। इसे बाद में ओपन-सोर्स बनाया गया था, और प्रधानमंत्री ने जुलाई 2021 में घोषणा की कि यह उन सभी देशों के लिए उपलब्ध है जो इसे चाहते थे। लेकिन प्रश्न इस तरह से तैयार किया गया है कि (b) को चिह्नित करना जोखिम भरा होगा, इसलिए (d) के साथ जाएं। हाँ, इसमें कुछ भ्रम है!

स्रोत: https://www.thehindu.com/sci-tech/technology/aarogya-setu-app-is-now-open-source-what-does-it-mean/article31689459.ece


12. वेब 3.0 के सन्दर्भ में, निम्नलिखित कथनों पर विचार कीजिए:

  1. वेब 3.0 प्रौद्योगिकी से व्यक्ति अपने स्वयं के आंकड़ों पर नियंत्रण कर सकते हैं।
  2. वेब 3.0 संसार में, ब्लॉकचेन आधारित सामाजिक नेटवर्क हो सकते हैं।
  3. वेब 3.0 किसी निगम द्वारा परिचालित होने की बजाय प्रयोक्ताओं द्वारा सामूहिक रूप से परिचालित किया जाता है। उपर्युक्त कथनों में कौन-से सही हैं?

  1. केवल 1 और 2
  2. केवल 2 और 3
  3. केवल 1 और 3
  4. 1, 2 और 3

Sol. Ans.(d). वेब 3.0, वेब/इंटरनेट के विकास के अगले पुनरावृत्ति या चरण का प्रतिनिधित्व करता है और वेब 2.0 के रूप में विघटनकारी साबित हो सकता है। वेब 3.0 विकेंद्रीकरण, खुलेपन और अधिक उपयोगकर्ता उपयोगिता की मूल अवधारणाओं पर बनाया गया है।

विकेंद्रीकरण: "वेब पर कुछ भी पोस्ट करने के लिए केंद्रीय प्राधिकरण से किसी अनुमति की आवश्यकता नहीं है, कोई केंद्रीय नियंत्रण नोड नहीं है, और इसलिए विफलता का कोई एकल बिंदु नहीं है ... और कोई 'किल स्विच' नहीं है! इसका मतलब अंधाधुंध सेंसरशिप और निगरानी से मुक्ति भी है।” यह वेब 3.0 का मुख्य सिद्धांत है। वेब 2.0 में, कंप्यूटर विशिष्ट वेब पतों के रूप में HTTP का उपयोग जानकारी खोजने के लिए करते हैं, जो एक निश्चित स्थान पर संग्रहीत होता है, आमतौर पर एक सर्वर पर। वेब 3.0 के साथ, क्योंकि इसकी सामग्री के आधार पर जानकारी मिलेगी, इसे एक साथ कई स्थानों पर संग्रहीत किया जा सकता है और इसलिए इसे विकेंद्रीकृत किया जा सकता है। यह वर्तमान में मेटा और गूगल जैसे इंटरनेट दिग्गजों के पास मौजूद विशाल डेटाबेस को तोड़ देगा और उपयोगकर्ताओं को अधिक नियंत्रण प्रदान करेगा। अतः 3 सत्य है।

बॉटम-अप डिज़ाइन: "विशेषज्ञों के एक छोटे समूह द्वारा कोड लिखे और नियंत्रित किए जाने के बजाय, इसे अधिकतम भागीदारी और प्रयोग को प्रोत्साहित करते हुए, सभी के पूर्ण दृष्टिकोण में विकसित किया गया था।"

वेब 3.0 के साथ, मोबाइल फोन, डेस्कटॉप, उपकरण, वाहन और सेंसर सहित असमान और तेजी से शक्तिशाली कंप्यूटिंग संसाधनों द्वारा उत्पन्न डेटा, उपयोगकर्ताओं द्वारा विकेन्द्रीकृत डेटा नेटवर्क के माध्यम से बेचा जाएगा, यह सुनिश्चित करते हुए कि उपयोगकर्ता स्वामित्व नियंत्रण बनाए रखते हैं । अतः 1 सत्य है।

भरोसेमंद और बिना अनुमति: विकेंद्रीकरण के अलावा और ओपन-सोर्स सॉफ़्टवेयर पर आधारित होने के कारण, वेब 3.0 भी भरोसेमंद होगा (यानी, नेटवर्क प्रतिभागियों को एक विश्वसनीय मध्यस्थ के बिना सीधे बातचीत करने की अनुमति देगा) और अनुमति रहित (जिसका अर्थ है कि कोई भी एक शासी निकाय से अनुमति बिना भाग ले सकता है)। नतीजतन, वेब 3.0 एप्लिकेशन ब्लॉकचैन या विकेन्द्रीकृत पीयर-टू-पीयर नेटवर्क, या उसके संयोजन पर चलेंगे-ऐसे विकेन्द्रीकृत ऐप्स को डीएपी कहा जाता है। अतः 2 सत्य है।

( यह प्रश्न सीधे पीटी के पावरपैक कोर्स में शामिल था - यहां नामांकन करें -https://bit.ly/upscias )

स्रोत: https://www.investopedia.com/web-20-web-30-5208698


13. “सॉफ्टवेयर, सेवा के रूप में (Software as a Service (Saas))" के सन्दर्भ में, निम्नलिखित कथनों पर विचार कीजिए:

  1. Saas क्रयकर्ता, प्रयोक्ता अन्तरापृष्ठ को अपनी आवश्यकतानुसार निर्धारित कर आंकड़ों के क्षेत्र में बदलाव कर सकते हैं।
  2. Saas प्रयोक्ता, अपनी चल युक्तियों (मोबाइल डिवाइसेज़) के माध्यम से अपने आंकड़ों तक पहुँच बना सकते हैं।
  3. आउटलुक, हॉटमेल और याहू! मेल Saas के रूप हैं।

उपर्युक्त कथनों में कौन-से सही हैं?

  1. केवल 1 और 2
  2. केवल 2 और 3
  3. केवल 1 और 3
  4. 1, 2 और 3

Sol. Ans.(d). SaaS एप्लिकेशन अक्सर अनुकूलन योग्य होते हैं और अन्य व्यावसायिक अनुप्रयोगों के साथ एकीकृत किए जा सकते हैं, विशेष रूप से एक सामान्य सॉफ्टवेयर प्रदाता के अनुप्रयोगों में स्पष्ट रूप से हम उनका उपयोग मोबाइल फोन पर कर सकते हैं । आउटलुक, हॉटमेल या याहू! मेल SaaS रूपों पर आधारित हैं।

स्रोत देखें - https://azure.microsoft.com/en-in/overview/what-is-saas/

( यह प्रश्न सीधे पीटी के पावरपैक कोर्स में शामिल था - यहां नामांकन करें -https://bit.ly/upscias )


14. निम्नलिखित कथनों में कौन-सा एक, जनसंचार माध्यमों में बहुचर्चित “प्रभाजी कक्षीय बमबारी प्रणाली" के आधारभूत विचार को सर्वोत्तम रूप से प्रतिबिंबित करता है।

  1. अंतरिक्ष में अतिध्वनिक मिसाइल का प्रमोचन, पृथ्वी की तरफ बढ़ते हुए क्षुद्रग्रह का सामना कर उसका अंतरिक्ष में ही विस्फोटन कराने के लिए किया जाता है।
  2. कोई अंतरिक्षयान अनेक कक्षीय गतियों के बाद किसी अन्य ग्रह पर उतरता है ।
  3. कोई मिसाइल पृथ्वी के परितः किसी स्थिर कक्षा में स्थापित किया जाता है और वह पृथ्वी पर किसी लक्ष्य के ऊपर कक्षा को त्यागता है।
  4. कोई अंतरिक्षयान किसी धूमकेतु के साथ-साथ उसी चाल से चलते हुए उसके पृष्ठ पर एक संपरीक्षित्र स्थापित करता है।

Sol. Ans.(c). एक एफओबीएस FOBS (प्रभाजी कक्षीय बमबारी प्रणाली) एक वॉरहेड डिलीवरी सिस्टम है जो अपने लक्ष्य गंतव्य की ओर पृथ्वी की स्थिर कक्षा का उपयोग करता है। लक्ष्य तक पहुँचने से ठीक पहले, यह एक प्रतिगामी इंजन बर्न से होकर गुजरता है। कभी इसे बहुत खतरनाक प्रणाली माना जाता था क्योंकि ऐसी मिसाइलों का पता लगाना बेहद मुश्किल होता था। उदाहरण के लिए, यह अपने लक्ष्य को मारने से पहले दक्षिणी ध्रुव के ऊपर की ओर यात्रा कर सकता है।

स्रोत: - https://en.wikipedia.org/wiki/Fractional_Orbital_Bombardment_System


15. “क्यूबिट (qubit)" शब्द का उल्लेख निम्नलिखित में कौन-से एक प्रसंग में होता है?

  1. क्लाउड सेवाएँ
  2. क्वांटम संगणन
  3. दृश्य प्रकाश संचार प्रौद्योगिकियाँ
  4. बेतार संचार प्रौद्योगिकियाँ

Sol. Ans.(b). यह एक बहुत ही आसान सवाल था। क्वांटम कंप्यूटिंग में, एक क्यूबिट या क्वांटम बिट क्वांटम सूचना की एक मूल इकाई है- क्लासिक बाइनरी बिट का क्वांटम संस्करण भौतिक रूप से दो- अवस्था डिवाइस के साथ महसूस किया जाता है।

( यह प्रश्न सीधे पीटी के पावरपैक कोर्स में शामिल था - यहां नामांकन करें -https://bit.ly/upscias )

स्रोत: - https://en.wikipedia.org/wiki/Qubit


16. निम्नलिखित संचार प्रौद्योगिकियों पर विचार कीजिए :

  1. निकट-परिपथ (क्लोज़-सर्किट) टेलीविज़न
  2. रेडियो आवृत्ति अभिनिर्धारण
  3. बेतार स्थानीय क्षेत्र नेटवर्क उपर्युक्त में कौन-सी लघु-परास युक्तियाँ/प्रौद्योगिकियाँ मानी जाती हैं?
  1. केवल 1 और 2
  2. केवल 2 और 3
  3. केवल 1 और 3
  4. 1, 2 और 3

Sol. Ans.(d). ये सभी शार्ट-रेंज यन्त्र ही हैं।

सीसीटीवी (निकट परिपथ टेलीविज़न) एक टीवी प्रणाली है जिसमें संकेतों को सार्वजनिक रूप से वितरित नहीं किया जाता है, लेकिन मुख्य रूप से निगरानी और सुरक्षा उद्देश्यों के लिए निगरानी की जाती है। सीसीटीवी कैमरों के रणनीतिक प्लेसमेंट और मॉनिटर पर कैमरे के इनपुट के निजी अवलोकन पर निर्भर करता है।

आरएफआईडी (रेडिओ आवृत्ति अभिनिर्धारण) एक ऐसी तकनीक है जो किसी टैग की गई वस्तु को निष्क्रिय रूप से पहचानने के लिए रेडियो तरंगों का उपयोग करती है। इसका उपयोग कई वाणिज्यिक और औद्योगिक अनुप्रयोगों में किया जाता है, जैसे की किसी आपूर्ति श्रृंखला के साथ वस्तुओं को ट्रैक करने से लेकर पुस्तकालय से बाहर की गई वस्तुओं का ट्रैक रखने तक।

बेतार स्थानीय क्षेत्र नेटवर्क एक वायरलेस कंप्यूटर नेटवर्क है जो एक सीमित क्षेत्र जैसे घर, स्कूल, कंप्यूटर प्रयोगशाला, परिसर, या कार्यालय भवन के भीतर एक स्थानीय क्षेत्र नेटवर्क बनाने के लिए वायरलेस संचार का उपयोग करके दो या दो से अधिक उपकरणों को जोड़ता है।

स्रोत: - https://microbewiki.kenyon.edu/index.php/Biofilms_and_Human_Implants


17. निम्नलिखित कथनों पर विचार कीजिए:

    1. जैवपरत (बायोफिल्म) मानव ऊतकों के भीतर चिकित्सकीय अंतर्रोपों पर बन सकती हैं!
    2. जैवपरत खाद्यपदार्थ और खाद्य प्रसंस्करण सतहों पर बन सकती हैं।
    3. जैवपरत प्रतिजैविक प्रतिरोध दर्शा सकती हैं।

    उपर्युक्त कथनों में कौन-से सही हैं?

    1. केवल 1 और 2
    2. केवल 2 और 3
    3. केवल 1 और 3
    4. 1, 2 और 3

Sol. Ans.(d). ) एक जैवपरत (बायोफिल्म) सतह से जुड़े माइक्रोबियल कोशिकाओं का एक संयोजन है जो एक बाह्य बहुलक पदार्थ मैट्रिक्स में संलग्न है। वैन लीउवेनहोएक ने अपने सरल सूक्ष्मदर्शी का उपयोग करते हुए, पहले दांतों की सतहों पर सूक्ष्मजीवों का अवलोकन किया और उन्हें माइक्रोबियल बायोफिल्म की खोज का श्रेय दिया जा सकता है। ये अक्सर प्रत्यारोपित चिकित्सा उपकरणों जैसे मूत्र और शिरापरक कैथेटर, स्तन प्रत्यारोपण और पेसमेकर की सतहों पर बनते हैं। अतः, 1 सही है। बायोफिल्म प्लास्टिक, कांच, धातु, सीमेंट, लकड़ी और खाद्य उत्पादों से लेकर खाद्य संयंत्रों में मौजूद सभी प्रकार की सतहों पर बनते हैं। अतः, 2 भी सही है।

बायोफिल्म संरचनाएं परिपक्व अवस्था में एंटीबायोटिक दवाओं के लिए अधिकतम प्रतिरोध दिखाती हैं। अतः, 3 भी सही है।

स्रोत 1 - https://www.intechopen.com/chapters/70036

स्रोत 2- https://www.tandfonline.com/doi/full/10.4161/viru.23724


18. प्रजैविकों (प्रोबायोटिक्स) के संदर्भ में निम्नलिखित कथनों पर विचार कीजिए:

  1. प्रजैविक, जीवाणु और यीस्ट दोनों के बने होते हैं।
  2. प्रजैविकों में जीव, खाए जाने वाले खाद्य में होते हैं किन्तु वे नैसर्गिक रूप से हमारी आहार-नली में नहीं पाए जाते।
  3. प्रजैविक दुग्ध शर्कराओं के पाचन में सहायक हैं।

उपर्युक्त कथनों में कौन-सा/से सही है/हैं?

  1. केवल 1
  2. केवल 2
  3. 1 और 3
  4. 2 और 3

Sol. Ans.(c). प्रजैविक (प्रोबायोटिक्स) जीवित बैक्टीरिया और/या खमीर का मिश्रण है जो आपके शरीर में रहता है। प्रोबायोटिक्स अच्छे बैक्टीरिया होते हैं जो आपको स्वस्थ रखने में मदद करते हैं। अतः, 1 सही है, विकल्प (b) और (c) खारिज है। कई प्रोबायोटिक जीवाणु उपभेद भी बीटा-गैलेक्टोसिडेज़ का उत्पादन करते हैं, एक एंजाइम जो लैक्टेज की तरह काम करता है और आंत में लैक्टोज को अतःड़ने में मदद कर सकता है। अतः, 3 भी सही है।

स्रोत 1 - https://my.clevelandclinic.org/health/articles/14598-probiotics

स्रोत 2 -
https://www.optibacprobiotics.com/professionals/latest-research/general-health/probiotic-supplements-help-lactose-intolerance#:~:text=But%20it's%20also%20the%20action,like%20lactase%20and%20can%20help


19. कोविड-19 विश्वमहामारी को रोकने के लिए बनाई जा रही वैक्सीनों के प्रसंग में, निम्नलिखित कथनों पर विचार कीजिए:

  1. भारतीय सीरम संस्थान ने mRNA प्लेटफॉर्म का प्रयोग कर कोविशील्ड नामक कोविड-19 वैक्सीन निर्मित की।
  2. स्पुतनिक v वैक्सीन रोगवाहक (वेक्टर) आधारित प्लेटफॉर्म का प्रयोग कर बनाई गई है।
  3. कोवैक्सीन एक निष्कृत रोगजनक आधारित वैक्सीन है।

उपर्युक्त कथनों में कौन-से सही हैं?

  1. केवल 1 और 2
  2. केवल 2 और 3
  3. केवल 1 और 3
  4. 1, 2 और 3

Sol. Ans.(b). ऑक्सफोर्ड-एस्ट्राजेनेका COVID-19 वैक्सीन, कोविशील्ड और वैक्सजेवरिया ब्रांड नामों के तहत बेचा जाता है, यह कोविड-19 की रोकथाम के लिए एक वायरल वेक्टर वैक्सीन है। अतः, कथन 1 सही नहीं है, और केवल संभावित विकल्प (b) है। बाकि तीन विकल्पों को खारिज किया जा सकता है। हम इस प्रश्न को आसान (sitter) मान सकते हैं!

( यह प्रश्न सीधे पीटी के पावरपैक कोर्स में शामिल था - यहां नामांकन करें -https://bit.ly/upscias )


20. यदि कोई मुख्य सौर तूफ़ान (सौर प्रज्वाल) पृथ्वी पर पहुँचता है, तो पृथ्वी पर निम्नलिखित में कौन-से संभव प्रभाव होंगे?

  1. GPS और दिक्संचालन (नैविगेशन) प्रणालियाँ विफल हो सकती हैं।
  2. विषुवतीय क्षेत्रों में सुनामियाँ आ सकती हैं ।
  3. बिजली ग्रिड क्षतिग्रस्त हो सकते हैं ।
  4. पृथ्वी के अधिकांश हिस्से पर तीव्र ध्रुवीय ___ज्योतियाँ घटित हो सकती हैं।
  5. ग्रह के अधिकांश हिस्से पर दावाग्नियाँ घटित हो सकती हैं।
  6. उपग्रहों की कक्षाएं विक्षुब्ध हो सकती हैं ।
  7. ध्रुवीय क्षेत्रों के ऊपर से उड़ते हुए वायुयान का लघुतरंग रेडियो संचार बाधित हो सकता है ।

नीचे दिए कूट का प्रयोग कर सही उत्तर चुनिए:

  1. केवल 1, 2, 4 और 5
  2. केवल 2, 3, 5, 6 और 7
  3. केवल 1, 3, 4, 6 और 7
  4. 1, 2, 3, 4, 5, 6 और 7

Sol. Ans.(c). सुनामी प्लेट टेक्टोनिक्स हलचल के कारण होती है। पृथ्वी से टकराने वाले सौर ज्वालाओं की घटनाओं के परिणामस्वरूप सुनामी नहीं आएगी क्योंकि वे दोनों परस्पर जुड़े नहीं हैं। अतः कथन 2 सही विकल्प का भाग नहीं होना चाहिए। केवल संभव विकल्प (c) है। यदि आप इस तथ्य को जानते अतः यह आसान था।

( यह प्रश्न सीधे पीटी के पावरपैक कोर्स में शामिल था - यहां नामांकन करें -https://bit.ly/upscias )

स्रोत: https://www.space.com/solar-flares-effects-classification-formation



21. निम्नलिखित कथनों पर विचार कीजिए:

  1. एच.एन. सान्याल समिति की रिपोर्ट के अनुसरण में, न्यायालय की अवमानना अधिनियम, 1971 पारित किया गया था।
  2. भारत का संविधान उच्चतम न्यायालय और उच्च न्यायालयों को, अपनी अवमानना के लिए दंड देने हेतु, शक्ति प्रदान करता है।
  3. भारत का संविधान सिविल अवमानना और आपराधिक अवमानना को परिभाषित करता है।
  4. भारत में, न्यायालय की अवमानना के विषय में कानून बनाने के लिए संसद में शक्ति निहित है।

उपर्युक्त कथनों में कौन-सा/से सही है/हैं ?

  1. केवल 1 और 2
  2. 1, 2 और 4
  3. केवल 3 और 4
  4. केवल 3

Sol. Ans.(a). 1961 में, भारत सरकार के लिए एक अतिरिक्त सॉलिसिटर जनरल, एच.एन. सान्याल की अध्यक्षता में एक समिति को भारत में अवमानना कानूनों के आवेदन की जांच के लिए नियुक्त किया गया था। इस समिति की अनुशंसा पर न्यायालय अवमानना अधिनियम, 1971 पारित किया गया। अतः 1 सही है।

कथन 3 पर ध्यान दें। यह गलत है, क्योंकि इन दो प्रकारों को 1971 के अधिनियम द्वारा परिभाषित किया गया था न कि संविधान द्वारा। अतः (a) या (b) हमारा जवाब हो सकता है।

अंतिम उत्तर है (a)।

स्रोत 1: - https://www.legalserviceindia.com/article/l255-Contempt-of-Court.html

स्रोत 2:-
https://www.jagranjosh.com/general-knowledge/explained-power-to-punish-for-contempt-of-court-cannot-be-abridged-by-parliament-sc-1633681057-1

स्रोत 3: - https://indianculture.gov.in/report-committee-contempt-courts-1963


22. भारत के सन्दर्भ में, निम्नलिखित कथनों पर विचार कीजिए:

  1. सरकारी विधि अधिकारी और विधिक फर्म ___ अधिवक्ता के रूप में मान्यता प्राप्त हैं, किन्तु कॉर्पोरेट वकील और पेटेंट न्यायवादी अधिवक्ता की मान्यता से बाहर रखे गए हैं।
  2. विधिज्ञ परिषदों (बार कौंसिलों) को विधिक शिक्षा और विधि महाविद्यालयों की मान्यता के बारे में नियम अधिकथित करने की शक्ति है।

उपर्युक्त कथनों में कौन-सा/से सही है/हैं?

  1. केवल 1
  2. केवल 2
  3. 1 और 2 दोनों
  4. न तो 1, न ही 2

Sol. Ans.(b). एक पेटेंट वकील एक अधिवक्ता है जो आईपी कानून का अभ्यास करता है और पेटेंट एजेंट हो भी सकता है और नहीं भी। इसके अलावा, एक पेटेंट वकील को अधिवक्ता के रूप में बार काउंसिल ऑफ इंडिया के साथ पंजीकृत होना चाहिए। एक अधिवक्ता भारतीय पेटेंट एजेंट परीक्षा लिखने के लिए पात्र नहीं है यदि वह वैज्ञानिक क्षेत्र से स्नातक नहीं है। (यह सच है कि कॉर्पोरेट वकीलों को अपना "सनद" समर्पण कर देना पड़ता है और वे अधिवक्ता के रूप में अभ्यास नहीं कर सकते हैं)

बार काउंसिल ऑफ इंडिया कानूनी शिक्षा को बढ़ावा देने और भारत में विश्वविद्यालयों और राज्य बार काउंसिल के परामर्श से मानकों को निर्धारित करने के अपने वैधानिक कार्य के हिस्से के रूप में देश में विश्वविद्यालयों / लॉ कॉलेजों का दौरा और निरीक्षण करता है।

अतः 1 सही नहीं है और 2 सही है।

स्रोत 1 -
http://www.barcouncilofindia.org/about/legal-education/#:~:text=The%20Bar%20Council%20of%20India%20visits%20and%20inspects%20Universities%2FLaw,and%20the%20State%20Bar%20Councils.

स्रोत 2-
https://www.origiin.com/2021/01/22/career-in-ipr-for-advocates/#:~:text=A%20patent%20attorney%20on%20the,graduate%20of%20a%20scientific%20domain.


23. निम्नलिखित कथनों पर विचार कीजिए:

  1. किसी संविधान संशोधन विधेयक को भारत के राष्ट्रपति की पूर्व सिफारिश की अपेक्षा होती है।
  2. जब कोई संविधान संशोधन विधेयक भारत के राष्ट्रपति के समक्ष प्रस्तुत किया जाता है, तो भारत के राष्ट्रपति के लिए यह बाध्यकर है कि वे अपनी अनुमति दें।
  3. संविधान संशोधन विधेयक लोक सभा और राज्य सभा दोनों द्वारा विशेष बहुमत से पारित होना ही चाहिए और इसके लिए संयुक्त बैठक का कोई उपबंध नहीं है।

उपर्युक्त कथनों में कौन-से सही हैं?

  1. केवल 1 और 2
  2. केवल 2 और 3
  3. केवल 1 और 3 ,
  4. 1, 2 और 3

Sol. Ans.(b). संविधान संशोधन प्रक्रिया अनुच्छेद 368 में निर्धारित है -

  1. संविधान में संशोधन केवल संसद के किसी भी सदन (लोकसभा और राज्य सभा) में इस उद्देश्य के लिए एक विधेयक पेश करके शुरू किया जा सकता है, न कि राज्य विधानसभाओं में।
  2. विधेयक को या अतः एक मंत्री या एक निजी सदस्य द्वारा पेश किया जा सकता है और इसके लिए राष्ट्रपति की पूर्व अनुमति की आवश्यकता नहीं होती है ।
  3. विधेयक को प्रत्येक सदन में विशेष बहुमत से पारित किया जाना चाहिए , यानी सदन की कुल सदस्यता का बहुमत (अर्थात 50 प्रतिशत से अधिक) और सदन के उपस्थित सदस्यों के दो-तिहाई बहुमत से और मतदान.
  4. प्रत्येक सदन को अलग-अलग विधेयक पारित करना होगा।
  5. दोनों सदनों के बीच असहमति के मामले में, विधेयक पर विचार-विमर्श और पारित होने के उद्देश्य से दोनों सदनों की संयुक्त बैठक आयोजित करने का कोई प्रावधान नहीं है ।
  6. यदि बिल संविधान के संघीय प्रावधानों में संशोधन करना चाहता है, अतः इसे आधे राज्यों के विधायिकाओं द्वारा साधारण बहुमत, यानी सदन के अधिकांश सदस्यों द्वारा उपस्थित और मतदान करने वाले सदस्यों द्वारा अनुमोदित किया जाना चाहिए।
  7. संसद के दोनों सदनों द्वारा विधिवत पारित होने और राज्य विधानसभाओं द्वारा अनुमोदित होने के बाद, जहां आवश्यक हो, विधेयक को राष्ट्रपति के पास सहमति के लिए प्रस्तुत किया जाता है।
  8. राष्ट्रपति को विधेयक पर अपनी सहमति देनी चाहिए। वह न अतः विधेयक पर अपनी सहमति रोक सकता है और न ही संसद के पुनर्विचार के लिए विधेयक को वापस कर सकता है।
  9. राष्ट्रपति की सहमति के बाद, बिल एक अधिनियम (यानी, एक संवैधानिक संशोधन अधिनियम) बन जाता है और संविधान अधिनियम की शर्तों के अनुसार संशोधित होता है।

अब ध्यान दें कि चूंकि कथन 1 गलत है, इसलिए तीन विकल्पों को खारिज कर दिया गया है।

( यह प्रश्न सीधे पीटी के पावरपैक कोर्स में शामिल था - यहां नामांकन करें -https://bit.ly/upscias )


24. निम्नलिखित कथनों पर विचार कीजिए:

  1. भारत का संविधान मंत्रियों को. चार श्रेणियों, अर्थात् कैबिनेट मंत्री, स्वतंत्र प्रभार वाले राज्यमंत्री, राज्यमंत्री और उपमंत्री, में वर्गीकृत करता है।
  2. संघ सरकार में मंत्रियों की कुल संख्या, प्रधान ___.मंत्री को मिला कर, लोक सभा के कुल सदस्यों के 15% से अधिक नहीं होनी चाहिए ।

उपर्युक्त कथनों में कौन-सा/से सही है/हैं?

  1. केवल 1
  2. केवल 2
  3. 1 और 2 दोनों
  4. न तो 1, न ही 2

Sol. Ans.(b). 91 वें संशोधन अधिनियम के अनुसार, मंत्रिपरिषद में प्रधान मंत्री सहित मंत्रियों की कुल संख्या लोक सभा के सदस्यों की कुल संख्या के पंद्रह प्रतिशत से अधिक नहीं होगी। इसलिए, (a) और (d) को खारिज किया जाता है। भारत का संविधान कैबिनेट मंत्री, स्वतंत्र प्रभारी राज्य मंत्री, राज्य मंत्री या उप मंत्री जैसे मंत्रियों के किसी भी रैंक को वर्गीकृत नहीं करता है। अत: केवल कथन 2 सही है। (वास्तव में, केवल तीन प्रकार के मंत्री होते हैं और "उप मंत्री" नामक कोई पद नहीं होता है)। अतः 1 गलत है।

( यह प्रश्न सीधे पीटी के पावरपैक कोर्स में शामिल था - यहां नामांकन करें -https://bit.ly/upscias )

स्रोत: https://legislative.gov.in/sites/default/files/amend91.pdf


25. निम्नलिखित में कौन-सी लोक सभा की अनन्य शक्तियाँ) है/हैं?

  1. आपात की उद्घोषणा का अनुसमर्थन करना
  2. मंत्रिपरिषद के विरुद्ध अविश्वास प्रस्ताव पारित करना
  3. भारत के राष्ट्रपति पर महाभियोग चलाना

नीचे दिए कूट का प्रयोग कर सही उत्तर चुनिए:

  1. 1 और 2
  2. केवल 2
  3. 1 और 3
  4. केवल 3

Sol. Ans.(b). लोकसभा के पास केवल कुछ शक्तियाँ हैं जो इसे राज्य सभा से अधिक शक्तिशाली बनाती हैं।

  1. सरकार के खिलाफ अविश्वास प्रस्ताव लोकसभा में पेश और पारित किया जा सकता है, लेकिन राज्यसभा में नहीं। यह सबसे महत्वपूर्ण अंतर है, क्योंकि मंत्रिपरिषद लोकसभा के प्रति उत्तरदायी है।
  2. धन विधेयक केवल लोकसभा में पेश किए जा सकते हैं
  3. भारत के राष्ट्रपति के नाम से वित्त मंत्री द्वारा लोकसभा में बजट पेश किया जाता है।
  4. गैर-वित्तीय (साधारण) विधेयकों के मामलों में, उस सदन द्वारा पारित किए जाने के बाद जहां इसे मूल रूप से पेश किया गया था (लोकसभा या राज्य सभा), इसे दूसरे सदन में भेजा जाता है, जहां इसे एक के लिए रखा जा सकता है 6 महीने की अधिकतम अवधि।
  5. संवैधानिक संशोधन (सदन की कुल सदस्यता के बहुमत और उपस्थित और मतदान करने वाले सदस्यों के कम से कम दो-तिहाई बहुमत से) के लिए किसी भी विधेयक को शुरू करने और पारित करने में राज्य सभा के समान अधिकार।
  6. राष्ट्रपति के महाभियोग (सदन की सदस्यता के दो-तिहाई) के लिए एक प्रस्ताव शुरू करने और पारित करने में राज्य सभा के समान अधिकार।
  7. उच्चतम न्यायालय और राज्य उच्च न्यायालयों के न्यायाधीशों (सदन की सदस्यता के बहुमत और कम से कम दो-तिहाई से) के महाभियोग की प्रक्रिया (हटाने के लिए एक प्रस्ताव शुरू करने और पारित करने) में राज्य सभा के समान अधिकार उपस्थित और मतदान करने वाले सदस्यों का बहुमत), जिन्हें तब भारत के राष्ट्रपति द्वारा हटाया जा सकता है।
  8. किसी राज्य में युद्ध या राष्ट्रीय आपातकाल (दो-तिहाई बहुमत से) या संवैधानिक आपातकाल (साधारण बहुमत से) घोषित करने वाले प्रस्ताव को शुरू करने और पारित करने में राज्य सभा के समान अधिकार।
  9. यदि राष्ट्रीय आपातकाल की घोषणा से पहले या बाद में लोकसभा भंग हो जाती है, अतः राज्य सभा एकमात्र संसद बन जाती है। इसे भंग नहीं किया जा सकता है। यह लोकसभा पर एक सीमा है।

( यह प्रश्न सीधे पीटी के पावरपैक कोर्स में शामिल था - यहां नामांकन करें -https://bit.ly/upscias )


26. भारत में दल-बदल विरोधी कानून के सन्दर्भ में, निम्नलिखित कथनों पर विचार कीजिए:

  1. यह कानून विनिर्दिष्ट करता है कि कोई नामनिर्दिष्ट विधायक सदन में नियुक्त होने के छह मास के अन्दर किसी राजनीतिक दल में शामिल नहीं हो सकता।
  2. यह कानून कोई समयावधि नहीं देता जिसके अन्दर पीठासीन अधिकारी को दल-बदल मामला विनिश्चित करना होता है।

उपर्युक्त कथनों में कौन-सा/से सही है/हैं?

  1. केवल 1
  2. केवल 2
  3. 1 और 2 दोनों
  4. न तो 1, न ही 2

Sol. Ans.(b). 15 फरवरी 1985 को दलबदल विरोधी कानून को राष्ट्रपति की मंजूरी मिली और अधिनियम 18 मार्च 1985 को लागू हुआ। कानून ने शेष कार्यकाल के लिए एक निर्वाचित सदस्य को अयोग्य घोषित करने की प्रक्रिया निर्धारित की, जिसने या तो इस्तीफा देकर या पार्टी नेतृत्व की अवहेलना करके और एक महत्वपूर्ण वोट पर अनुपस्थित रह कर दलबदल किया। दूसरे शब्दों में, दल-बदल विरोधी कानून संसद के व्यक्तिगत सदस्यों (सांसदों)/विधायकों को एक पार्टी को दूसरे के लिए छोड़ने के लिए दंडित करता है।

अयोग्यता के आधार

  1. यदि कोई निर्वाचित सदस्य स्वेच्छा से किसी राजनीतिक दल की सदस्यता छोड़ देता है।
  2. यदि वह पूर्व अनुमति प्राप्त किए बिना अपने राजनीतिक दल या ऐसा करने के लिए अधिकृत किसी व्यक्ति द्वारा जारी किसी भी निर्देश के विपरीत ऐसे सदन में मतदान करता है या मतदान से दूर रहता है।
  3. उसकी अयोग्यता के लिए पूर्व शर्त के रूप में, उसकी पार्टी या अधिकृत व्यक्ति द्वारा ऐसी घटना के 15 दिनों के भीतर मतदान से परहेज नहीं किया जाना चाहिए।
  4. यदि कोई स्वतंत्र रूप से निर्वाचित सदस्य किसी राजनीतिक दल में शामिल हो जाता है।
  5. यदि कोई मनोनीत सदस्य छह महीने की समाप्ति के बाद किसी राजनीतिक दल में शामिल होता है।

हालांकि, कानून कोई समयावधि प्रदान नहीं करता है जिसके भीतर पीठासीन अधिकारी को दलबदल मामला विनिश्चित करना होता है।


27. निम्नलिखित कथनों पर विचार कीजिए:

  1. भारत का महान्यायवादी और भारत का सॉलिसिटर जनरल ही सरकार के एकमात्र अधिकारी हैं जिन्हें भारत की संसद की बैठकों में भाग लेने की अनुमति है।
  2. भारत के संविधान के अनुसार, भारत का महान्यायवादी अपना त्यागपत्र दे देता है, जब वह सरकार जिसने उसको नियुक्त किया था इस्तीफा देती है।

उपर्युक्त कथनों में कौन-सा/से सही है/हैं?

  1. केवल 1
  2. केवल 2
  3. 1 और 2 दोनों
  4. न तो 1, न ही 2

Sol. Ans.(d). संविधान के अनुच्छेद 76 में उल्लेख है कि महान्यायवादी (अटॉर्नी जनरल) भारत का सर्वोच्च कानून अधिकारी है। भारत सरकार के मुख्य कानूनी सलाहकार के रूप में, वह सभी कानूनी मामलों पर केंद्र सरकार को सलाह देता है। वह भारत के सर्वोच्च न्यायालय में केंद्र सरकार का प्रतिनिधित्व करने वाले प्राथमिक वकील भी हैं। अटॉर्नी जनरल, एक राज्य के महाधिवक्ता की तरह, राजनीतिक रूप से नियुक्त व्यक्ति नहीं माना जाता है, लेकिन व्यवहार में ऐसा नहीं है।

भारत के राष्ट्रपति एक ऐसे व्यक्ति की नियुक्ति करते हैं जो सर्वोच्च न्यायालय के न्यायाधीश के पद के लिए योग्य हो। अटॉर्नी जनरल की नियुक्ति राष्ट्रपति द्वारा सरकार की सलाह पर की जाती है। निम्नलिखित योग्यताएं हैं - वह एक भारतीय नागरिक होना चाहिए; उन्होंने या अतः किसी भारतीय राज्य के उच्च न्यायालय में न्यायाधीश के रूप में 5 वर्ष या उच्च न्यायालय में अधिवक्ता के रूप में 10 वर्ष पूरे किए हों; राष्ट्रपति की नजर में वह एक प्रख्यात विधिवेत्ता भी हो सकते हैं।

भारत के महान्यायवादी के लिए कोई निश्चित अवधि नहीं है। संविधान में अटॉर्नी जनरल के किसी निर्दिष्ट कार्यकाल का उल्लेख नहीं है। इसी तरह, संविधान में भी उसे हटाने की प्रक्रिया और आधार का उल्लेख नहीं है ।

उन्हें राष्ट्रपति द्वारा किसी भी समय हटाया जा सकता है। वह केवल राष्ट्रपति को अपना त्यागपत्र देकर पद छोड़ सकता है। चूंकि उन्हें राष्ट्रपति द्वारा मंत्रिपरिषद की सलाह पर नियुक्त किया जाता है, पारंपरिक रूप से उन्हें (राष्ट्रपति द्वारा) हटा दिया जाता है जब परिषद भंग हो जाती है या बदल दी जाती है।

भारत का महान्यायवादी संसद के दोनों सदनों की बैठकों में भाग ले सकता है, जबकि वह दोनों में से किसी का भी सदस्य नहीं है ।

भारत का सॉलिसिटर जनरल भारत के महान्यायवादी के अधीन होता है। वह देश के दूसरे कानून अधिकारी हैं, और अटॉर्नी जनरल की सहायता करते हैं, और भारत के लिए अतिरिक्त सॉलिसिटर जनरल द्वारा सहायता प्रदान की जाती है। वे संसद की बैठकों में शामिल नहीं हो सकते हैं। अत: दोनों कथन सत्य नहीं हैं।


28. भारत के न्यायालयों द्वारा जारी रिटों के सन्दर्भ में, निम्नलिखित कथनों पर विचार कीजिए:

  1. किसी प्राइवेट संगठन के विरुद्ध, जब तक कि उसको कोई सार्वजनिक कार्य नहीं सौंपा गया हो, परमादेश (मैंडेमस) नहीं होगा।
  2. किसी कंपनी के विरुद्ध, भले ही वह कोई सरकारी कंपनी हो, परमादेश (मैंडेमस) नहीं होगा।
  3. कोई भी लोक-प्रवण व्यक्ति (पब्लिक माइंडेड परसन) अधिकार-पृच्छा (क्वो वारंटो) रिट प्राप्त करने हेतु न्यायालय में समावेदन करने के लिए याची (पिटीशनर) हो सकता है ।

उपर्युक्त कथनों में कौन-से सही हैं?

  1. केवल 1 और 2
  2. केवल 2 और 3
  3. केवल 1 और 3
  4. 1, 2 और 3

Sol. Ans.(c). 'परमादेश (मैंडमस)' का अर्थ है 'हम आज्ञा देते हैं'। यह एक अधिकार प्राप्त न्यायालय द्वारा एक सार्वजनिक प्राधिकरण को कानूनी कर्तव्यों का पालन करने के लिए निर्देशित करने के लिए जारी किया जाता है, जिसे उसने नहीं किया है या करने से इनकार कर दिया है। इसे न्यायालय द्वारा किसी सार्वजनिक अधिकारी, सार्वजनिक निगम, न्यायाधिकरण, अवर न्यायालय या सरकार के विरुद्ध जारी किया जा सकता है। इसे किसी निजी व्यक्ति या निकाय, राज्यों के राष्ट्रपति या राज्यपालों या एक कार्यरत मुख्य न्यायाधीशों के खिलाफ जारी नहीं किया जा सकता है।

इसे निम्नलिखित परिस्थितियों में जारी नहीं किया जा सकता है - विचाराधीन कर्तव्य विवेकाधीन है और अनिवार्य नहीं है। एक गैर-सांविधिक कार्य के प्रदर्शन के लिए। कर्तव्य के प्रदर्शन में विशुद्ध रूप से निजी प्रकृति के अधिकार शामिल हैं। जहां ऐसे निर्देश में किसी कानून का उल्लंघन शामिल हो।

संविधान के अनुच्छेद 226 के तहत, उक्त प्राधिकरण द्वारा किए जा रहे सार्वजनिक कार्यों के संबंध में एक निजी प्राधिकरण के खिलाफ भी परमादेश की एक रिट जारी की जा सकती है । रिट अदालत अपने निर्णय से एक निजी प्राधिकरण को सार्वजनिक कार्यों के निर्वहन को सही करने या लागू करने का निर्देश दे सकती है। अतः, 1 सही है।

सरकारी कंपनियों को परमादेश जारी करने को लेकर कानून असमंजस की स्थिति में नजर आ रहा है! कुछ उच्च न्यायालयों ने सोचा है कि रिट सरकारी कंपनी पर भी लागू हो सकती है, लेकिन कुछ अन्य लोगों ने माना है कि जीवन बीमा निगम जैसे वैधानिक निगम को रिट जारी नहीं किया जा सकता है। उपरोक्त निकायों के संबंध में स्थिति की जांच सैरएटिम (एक लैटिन शब्द जिसका अर्थ लगातार होता है ) की जाती है।

अधिकार-प्रच्छः (क्वो वॉरन्टो) की रिट अदालतों द्वारा किसी निजी व्यक्ति के खिलाफ जारी की जाती है जब वह एक ऐसा पद ग्रहण करता है जिस पर उसका कोई अधिकार नहीं होता है। अधिकार-प्रच्छः का शाब्दिक अर्थ है 'किस अधिकार से' और यह लोगों को सार्वजनिक कार्यालयों पर कब्जा करने से रोकने के लिए एक प्रभावी उपाय है। एक नागरिक अधिकार-प्रच्छः के रिट का दावा कर सकता है और वह एक संबंधक की स्थिति में खड़ा होता है। उसे किसी विशेष रुचि या व्यक्तिगत रुचि की आवश्यकता नहीं है।

स्रोत 1: https://www.jstor.org/stable/43950186

स्रोत 2: https://indiankanoon.org/doc/1760972/#:~:text=Under%20the%20said%20Article%2C%20a,enforce%20discharge%20of%20public%20functions

स्रोत 3: https://main.sci.gov.in/pdf/SupremeCourtReport/2012_v10_piv.pdf


29. आयुष्मान भारत डिजिटल मिशन के सन्दर्भ में, निम्नलिखित कथनों पर विचार कीजिए:

  1. प्राइवेट अस्पतालों और सरकारी अस्पतालों को इसे अवश्य अपनाना चाहिए।
  2. चूँकि इसका लक्ष्य स्वास्थ्य की सर्वजनीन व्याप्ति है, अंततोगत्वा भारत के हर नागरिक को इसका हिस्सा हो जाना चाहिए।
  3. यह पूरे देश में निर्बाध रूप से लागू किया जा सकता है । उपर्युक्त कथनों में कौन-सा/से सही है/हैं?

उपर्युक्त कथनों में कौन-सा/से सही है/हैं?

  1. केवल 1 और 2
  2. केवल 3
  3. केवल 1 और 3
  4. 1, 2 और 3

Sol. Ans.(b). आयुष्मान भारत डिजिटल मिशन (एबीडीएम) का उद्देश्य सभी भारतीय नागरिकों के लिए डिजिटल स्वास्थ्य आईडी प्रदान करना है ताकि अस्पतालों, बीमा फर्मों और नागरिकों को आवश्यकता पड़ने पर इलेक्ट्रॉनिक रूप से स्वास्थ्य रिकॉर्ड तक पहुंचने में मदद मिल सके। प्रत्येक नागरिक के लिए एक हेल्थ आईडी जारी की जाएगी जो उनके स्वास्थ्य खाते के रूप में भी काम करेगी। इस स्वास्थ्य खाते में हर परीक्षण, हर बीमारी, डॉक्टर के पास जाना, ली गई दवाओं और निदान का विवरण होगा। स्वास्थ्य आईडी निःशुल्क है, स्वैच्छिक है। यह स्वास्थ्य डेटा का विश्लेषण करने में मदद करेगा और स्वास्थ्य कार्यक्रमों के लिए बेहतर योजना, बजट और कार्यान्वयन की ओर ले जाएगा।

कार्यक्रम का अन्य प्रमुख घटक एक हेल्थकेयर प्रोफेशनल्स रजिस्ट्री (HPR) और हेल्थकेयर फैसिलिटीज रजिस्ट्री (HFR) बनाना है, जिससे मेडिकल प्रोफेशनल्स और हेल्थ इंफ्रास्ट्रक्चर तक आसान इलेक्ट्रॉनिक एक्सेस की अनुमति मिलती है। एचपीआर चिकित्सा की आधुनिक और पारंपरिक दोनों प्रणालियों में स्वास्थ्य सेवा प्रदान करने में शामिल सभी स्वास्थ्य पेशेवरों का एक व्यापक भंडार होगा। एचएफआर डेटाबेस में देश की सभी स्वास्थ्य सुविधाओं (सार्वजनिक और निजी) का रिकॉर्ड होगा।

एबीडीएम में भागीदारी नागरिकों सहित स्वैच्छिक है। स्वास्थ्य सुविधा या संस्थान की भागीदारी भी स्वैच्छिक है और संबंधित प्रबंधन (सरकारी या निजी प्रबंधन) द्वारा ली जाएगी। हालांकि, एक बार जब प्रबंधन संबंधित स्वास्थ्य सुविधा/संस्थान को एबीडीएम में पंजीकृत करने का निर्णय लेता है, तो उक्त सुविधा/संस्थान की सेवा करने वाले सभी स्वास्थ्य पेशेवरों के लिए हेल्थकेयर प्रोफेशनल्स रजिस्ट्री में पंजीकरण करना आवश्यक है ताकि संस्थान पूरी तरह से राष्ट्रीय डिजिटल स्वास्थ्य (एनडीएचई) के पारिस्थितिकी तंत्र साथ एकीकृत हो सके।

अतः, 1 सही नहीं है, केवल संभावित विकल्प (b) है।

स्रोत: https://abdm.gov.in/faq


30. लोकसभा के उपाध्यक्ष के सन्दर्भ में, निम्नलिखित कथनों पर विचार कीजिए:

  1. लोकसभा के कार्य-पद्धति और कार्य संचालन नियमों के अनुसार, उपाध्यक्ष का निर्वाचन उस तारीख को होगा जो अध्यक्ष नियत करे।
  2. यह आज्ञापक उपबंध है कि लोकसभा के उपाध्यक्ष के रूप में किसी प्रतियोगी का निर्वाचन या तो मुख्य विपक्षी दल से, या शासक दल से, होगा।
  3. सदन की बैठक की अध्यक्षता करते समय उपाध्यक्ष की शक्ति वैसी ही होती है जैसी कि अध्यक्ष की, और उसके विनिर्णयों के विरुद्ध कोई अपील नहीं हो सकती।
  4. उपाध्यक्ष की नियुक्ति के बारे में सुस्थापित संसदीय - पद्धति यह है कि प्रस्ताव अध्यक्ष द्वारा रखा जाता है __ और प्रधान मंत्री द्वारा विधिवत समर्थित होता

उपर्युक्त कथनों में कौन-से सही हैं?

  1. केवल 1 और 3
  2. 1, 2 और 3
  3. केवल 3 और 4
  4. केवल 2 और 4

Sol. Ans.(a). लोकसभा के उपाध्यक्ष लोकसभा के दूसरे सर्वोच्च रैंकिंग विधायी अधिकारी हैं, जो भारत की संसद के निचले सदन हैं।

लोकसभा अध्यक्ष का चुनाव होने के ठीक बाद उपाध्यक्ष का चुनाव भी लोकसभा द्वारा अपने सदस्यों में से किया जाता है। उपाध्यक्ष के चुनाव की तिथि अध्यक्ष द्वारा निर्धारित की जाती है (अध्यक्ष के चुनाव की तिथि राष्ट्रपति द्वारा निर्धारित की जाती है)। अतः, 1 सही है।

उपाध्यक्ष अध्यक्ष के कार्यालय के रिक्त होने पर उसके कर्तव्यों का पालन करता है। वह अध्यक्ष के रूप में भी कार्य करता है जब वह सदन की बैठक से अनुपस्थित रहता है । यदि अध्यक्ष ऐसी बैठक से अनुपस्थित रहता है तो वह संसद के दोनों सदनों की संयुक्त बैठक की अध्यक्षता भी करता है। उपाध्यक्ष के पास एक विशेष विशेषाधिकार होता है, अर्थात जब भी उसे संसदीय समिति के सदस्य के रूप में नियुक्त किया जाता है, अतः वह स्वतः ही उसका अध्यक्ष बन जाता है। अतः, 3 सही है।

1 और 3 सही हैं - इसलिए केवल संभव विकल्प (a) है।


31. “त्वरित वित्तीयन प्रपत्र (Rapid Financing Instrument)” और “त्वरित ऋण सुविधा (Rapid Credit Facility)", निम्नलिखित में किस एक के द्वारा उधार दिए जाने के उपबंधों से संबंधित हैं?

  1. एशियाई विकास बैंक
  2. अंतर्राष्ट्रीय मुद्रा कोष
  3. संयुक्त राष्ट्र पर्यावरण कार्यक्रम वित्त पहल
  4. विश्व बैंक

Sol. Ans.(b). सबसे पहले, त्वरित वित्तीयन प्रपत्र (रैपिड फाइनेंसिंग इंस्ट्रूमेंट - आरएफआई) - यह तेजी से वित्तीय सहायता प्रदान करता है, जो भुगतान संतुलन की तत्काल आवश्यकता का सामना करने वाले सभी सदस्य देशों के लिए उपलब्ध है। इसे सदस्य देशों की विविध जरूरतों को पूरा करने के उद्देश्य से आईएमएफ की वित्तीय सहायता को और अधिक लचीला बनाने के लिए व्यापक सुधार के हिस्से के रूप में बनाया गया था। आरएफआई ने आईएमएफ की पिछली आपातकालीन सहायता नीति को बदल दिया है और इसे कई तरह की परिस्थितियों में इस्तेमाल किया जा सकता है।

त्वरित ऋण सुविधा (रैपिड क्रेडिट सुविधा - आरसीएफ) - यह कम आय वाले देशों (एलआईसी) को बिना किसी पूर्व शर्त के तत्काल रियायती वित्तीय सहायता प्रदान करती है, जो भुगतान संतुलन (बीओपी) की समस्या का सामना कर रहे हैं, जहां एक पूर्ण आर्थिक कार्यक्रम न तो आवश्यक है और न ही संभव है। आरसीएफ को गरीबी में कमी और विकास ट्रस्ट (पीआरजीटी) के तहत एक व्यापक सुधार के हिस्से के रूप में बनाया गया था ताकि फंड की वित्तीय सहायता को अधिक लचीला और एलआईसी की विविध आवश्यकताओं के अनुरूप बनाया जा सके, जिसमें संकट के समय भी शामिल है।

स्रोत: https://www.imf.org/en/About/Factsheets/Sheets/2016/08/02/19/55/Rapid-Financing-Instrument

( यह प्रश्न सीधे पीटी के पावरपैक कोर्स में शामिल था - यहां नामांकन करें -https://bit.ly/upscias )


32. भारतीय अर्थव्यवस्था के सन्दर्भ में, निम्नलिखित कथनों पर विचार कीजिए:

  1. अंकित प्रभावी विनिमय दर (Nominal Effective Exchange Rate (NEER)) में वृद्धि रुपए की मूल्यवृद्धि को दर्शाता है।
  2. वास्तविक प्रभावी विनिमय दर (Real Effective Exchange Rate (REER) में वृद्धि व्यापार प्रतिस्पर्धात्मकता में सुधार को दर्शाता है।
  3. अन्य देशों में मुद्रास्फीति के सापेक्ष घरेलू मुद्रास्फीति में बढ़ने की प्रवृत्ति NEER और REER के बीच में वर्धमान अपसरण उत्पन्न कर सकता है।

उपर्युक्त कथनों में कौन-से सही हैं?

  1. केवल 1 और 2
  2. केवल 2 और 3
  3. केवल 1 और 3
  4. 1, 2 और 3

Sol. Ans.(c). एनईईआर (अंकित प्रभावी विनिमय दर) कई विदेशी मुद्रा के भारित औसत के मुकाबले मुद्रा के मूल्य का एक उपाय है। एनईईआर में वृद्धि रुपये की सराहना का संकेत देती है। अत: कथन 1 सही है।

आरईईआर में वृद्धि का अर्थ है कि निर्यात अधिक महंगा हो जाता है और आयात सस्ता हो जाता है; इसलिए, वृद्धि व्यापार प्रतिस्पर्धा में कमी का संकेत देती है। अतः, कथन 2 गलत है।

एनईईआर विदेशी मुद्राओं के संदर्भ में घरेलू मुद्रा की द्विपक्षीय नाममात्र विनिमय दरों का भारित गुणोत्तर माध्य है। आरईईआर एनईईआर का भारित औसत है जिसे घरेलू कीमत और विदेशी कीमतों के अनुपात से समायोजित किया जाता है। अन्य देशों में मुद्रास्फीति के सापेक्ष घरेलू मुद्रास्फीति की बढ़ती प्रवृत्ति एनईईआर और आरईईआर में अंतर पैदा करती है। अतः, 3 सही है।

यह एक बहुत ही कठिन प्रश्न था, और इसे तब तक टाला जा सकता था जब तक कि पहले अच्छी तरह से अध्ययन न किया गया हो।

( यह प्रश्न सीधे पीटी के पावरपैक कोर्स में शामिल था - यहां नामांकन करें -https://bit.ly/upscias )


33. भारतीय अर्थव्यवस्था के सन्दर्भ में, निम्नलिखित कथनों पर विचार कीजिए:

  1. यदि मुद्रास्फीति अत्यधिक है, तो भारतीय रिजर्व बैंक (RBI) सम्भावित रूप से सरकारी प्रतिभूतियाँ खरीद सकता है।
  2. यदि रुपए का तेजी से मूल्यह्रास हो रहा है, तो RBI बाज़ार में डॉलरों का सम्भावित रूप से विक्रय कर सकता है।
  3. यदि USA या यूरोपीय संघ में ब्याज दरें गिरती होतीं, तो इससे सम्भावित रूप से RBI की डॉलरों की खरीद प्रेरित हो सकती है ।

उपर्युक्त कथनों में कौन-से सही हैं?

  1. केवल 1 और 2
  2. केवल 2 और 3
  3. केवल 1 और 3
  4. 1, 2 और 3

Sol. Ans.(b). जब मुद्रास्फीति अधिक होती है तो आरबीआई खुले बाजार संचालन (ओएमओ) के माध्यम से सरकारी प्रतिभूतियों को जनता को बेचकर बाजार से तरलता को कम करने का प्रयास करता है। अतः, कथन 1 गलत है। अतः विकल्प (a), (c) और (d) ख़ारिज किये जा सकते है। उत्तर होना चाहिए (b), अतः यह एक सिटर है!

अब - "रुपये का मूल्यह्रास" का अर्थ डॉलर के संबंध में रुपये के मूल्य में कमी है। एक मुक्त अस्थायी विनिमय दर व्यवस्था में, मूल्यह्रास तब होता है जब डॉलर की मांग आपूर्ति से अधिक होती है। डॉलर की आपूर्ति बढ़ाने के लिए (और तेजी से मूल्यह्रास को रोकने के लिए) अगर रुपये में तेजी से गिरावट आती है, अतः आरबीआई बाजार में डॉलर बेचने की संभावना रखता है। अत: कथन 2 सही है।

जब अमेरिका और यूरोपीय संघ में ब्याज दरें गिरती हैं, तो भारतीय बाजार से डॉलर की आमद होगी ("विदेशों में सस्ते पैसे का प्रभाव"), जिससे रुपये के मूल्य में वृद्धि होगी। इससे भारतीय निर्यातकों और आईटी फर्मों (और विदेशों में भी एनआरआई!) को नुकसान होगा। अर्थव्यवस्था में डॉलर की आपूर्ति कम करने के लिए आरबीआई बाजार से डॉलर खरीदेगा। अत: कथन 3 भी सही है।

( यह प्रश्न सीधे पीटी के पावरपैक कोर्स में शामिल था - यहां नामांकन करें -https://bit.ly/upscias )


34. “G20 कॉमन फ्रेमवर्क" के सन्दर्भ में, निम्नलिखित कथनों पर विचार कीजिए:

  1. यह G20 और उसके साथ पेरिस क्लब द्वारा समर्थित पहल है।
  2. यह अधारणीय ऋण वाले निम्न आय देशों को सहायता देने की पहल है।

उपर्युक्त कथनों में कौन-सा/से सही है/हैं?

  1. केवल 1
  2. केवल 2
  3. 1 और 2 दोनों
  4. न तो 1, न ही 2

Sol. Ans.(c). पेरिस क्लब के साथ मिलकर जी20 द्वारा समर्थित पहल और कम आय वाले देशों को अस्थिर ऋण के साथ समर्थन करने की एक पहल है। दोनों कथन सही हैं। कृपया स्रोत अधिक जानकारी देखें।

स्रोत: https://blogs.imf.org/2021/12/02/the-g20-common-framework-for-debt-treatments-must-be-stepped-up/


35. भारतीय अर्थव्यवस्था के सन्दर्भ में, "मुद्रास्फीति-संहलग्न बॉन्ड (Inflation-Indexed Bonds (IIBs))” के क्या लाभ हैं?

  1. सरकार IIBS के रूप में अपने ऋणग्रहण पर कूपन दरों को कम कर सकती है।
  2. IIBs निवेशकों को मुद्रास्फीति के बारे में अनिश्चितता से सुरक्षा प्रदान करते हैं।
  3. IIBS पर प्राप्त ब्याज और साथ ही साथ पूँजीगत लाभ कर-योग्य नहीं होते।

उपर्युक्त कथनों में कौन-से सही हैं?

  1. केवल 1 और 2
  2. केवल 2 और 3
  3. केवल 1 और 3
  4. 1, 2 और 3

Sol. Ans.(a). सरकार आईआईबी के माध्यम से अपने उधार पर कूपन दरों को कम कर सकती है। मुद्रास्फीति सहलग्न बांड निवेशकों को मुद्रास्फीति के संबंध में अनिश्चितता से सुरक्षा प्रदान करते हैं। मौजूदा कर प्रावधान आईआईबी पर ब्याज भुगतान और पूंजीगत लाभ पर लागू होंगे। अतः 1 और 2 सही हैं और 3 नहीं। (इस प्रश्न के लिए एक छात्र को कई प्रकार के वित्तीय साधनों के बारे में अच्छी तरह से जानना आवश्यक है)।

स्रोत : https://m.rbi.org.in/Scripts/FAQView.aspx?Id=91


36. भारत में कार्य कर रही विदेशी-स्वामित्व की e-वाणिज्य फर्मों के सन्दर्भ में, निम्नलिखित कथनों में कौन-सा/से सही है/हैं?

  1. अपने प्लेटफॉर्मों को बाज़ार-स्थान के रूप में प्रस्तुत करने के अतिरिक्त वे स्वयं अपने माल का विक्रय भी कर सकते हैं।
  2. वे अपने प्लेटफॉर्मों पर किस अंश तक बड़े विक्रेताओं को स्वीकार कर सकते हैं, यह सीमित

नीचे दिए कूट का प्रयोग कर सही उत्तर चुनिए:

  1. केवल 1
  2. केवल 2
  3. 1 और 2 दोनों
  4. न तो 1, न ही 2

Sol. Ans.(b). "क्या एफडीआई-वित्त पोषित ई-कॉमर्स फर्म अपने स्वयं के मार्केटप्लेस पर इन्वेंट्री रख सकते हैं या नहीं" के मुद्दे ने पिछले कुछ वर्षों में कई नीति फ्लिप-फ्लॉप देखे हैं।

भारत ने हाल ही में ऑनलाइन मार्केटप्लेस द्वारा स्थापित कंपनियों को अपने प्लेटफॉर्म पर ट्रेडिंग करने से रोकने के लिए ई-कॉमर्स के लिए प्रत्यक्ष विदेशी निवेश (एफडीआई) नियमों को कड़ा किया है। नियमों ने ऑनलाइन मार्केटप्लेस को अपनी खुद की इन्वेंट्री रखने या अपने प्लेटफॉर्म पर माल की कीमत को प्रभावित करने से रोक दिया। उन्होंने, अन्य बातों के अलावा, समूह कंपनियों या संस्थाओं को भी प्रतिबंधित कर दिया, जिनमें मार्केटप्लेस का अपने प्लेटफॉर्म पर बेचने के लिए इन्वेंट्री का नियंत्रण होता है। (इनमें से कई नियम कई सालों से मौजूद हैं)

भारत में एफडीआई नीति के तहत, ई-कॉमर्स गतिविधियों पर इक्विटी/एफडीआई कैप स्वचालित मार्ग के माध्यम से 100% निर्धारित की गई है। हालांकि, ई-कॉमर्स स्टार्टअप और संस्थाओं को केवल बिजनेस टू बिजनेस (बी2बी) ई-कॉमर्स में संलग्न होना चाहिए, न कि बिजनेस टू कंज्यूमर (बी2सी) ई-कॉमर्स में। (ऐसा करना वास्तव में फ्लिपकार्ट या अमेज़ॅन जैसी फर्मों के लिए कोई मतलब नहीं होगा!)

एफडीआई नीति ई-कॉमर्स गतिविधियों के मार्केटप्लेस मॉडल के लिए स्वचालित मार्ग के तहत 100% एफडीआई की अनुमति देती है। हालांकि, ई-कॉमर्स गतिविधियों के इन्वेंट्री-आधारित मॉडल के लिए एफडीआई की अनुमति नहीं है।

ई-कॉमर्स के बाज़ार-आधारित मॉडल का अर्थ है एक डिजिटल और इलेक्ट्रॉनिक नेटवर्क पर ई-कॉमर्स स्टार्टअप या इकाई द्वारा एक सूचना प्रौद्योगिकी मंच प्रदान करना, जो खरीदार और विक्रेता के बीच एक सुविधा के रूप में कार्य करता है।

ई-कॉमर्स के इन्वेंट्री-आधारित मॉडल का मतलब ई-कॉमर्स गतिविधियों से है जहां वस्तुओं और सेवाओं की सूची का स्वामित्व ई-कॉमर्स स्टार्टअप या इकाई के पास होता है और इसे सीधे उपभोक्ताओं को बेचा जाता है।

अतः, कथन 1 सही नहीं है, लेकिन 2 सही है।

स्रोत 1:
https://economictimes.indiatimes.com/tech/tech-bytes/2021-year-in-review-the-chaos-and-confusion-around-indias-ecommerce-rules/articleshow/88614007.cms?from=mdr

स्रोत 2: https://cleartax.in/s/fdi-regulations-e-commerce-startups


37. निम्नलिखित में कौन-कौन-से कार्यकलाप अर्थव्यवस्था में वास्तविक क्षेत्रक (रियल सेक्टर) का निर्माण करते

  1. किसानों का अपनी फ़सलें काटना
  2. कपड़ा मिलों का कच्चे कपास को कपड़े में बदलना
  3. किसी वाणिज्यिक बैंक का किसी व्यापारी कंपनी को धनराशि उधार देना
  4. किसी कॉर्पोरेट निकाय का विदेश में - रुपया-अंकित मूल्य के बॉन्ड जारी करना

नीचे दिए कूट का प्रयोग कर सही उत्तर चुनिए:

  1. केवल 1 और 2
  2. केवल 2, 3 और 4
  3. केवल 1, 3 और 4
  4. 1, 2, 3 और 4

Sol. Ans.(a). अर्थव्यवस्था के वास्तविक क्षेत्र में उद्यम (गैर-वित्तीय निगम), परिवार और गैर-लाभकारी संस्थाएं हैं जो परिवारों की सेवा करती हैं। विवरण 3 और 4 में वित्तीय निगमों का उल्लेख है, और इसलिए उन्हें अर्थव्यवस्था में वास्तविक क्षेत्र के रूप में नहीं माना जा सकता है।

( यह प्रश्न सीधे पीटी के पावरपैक कोर्स में शामिल था - यहां नामांकन करें -https://bit.ly/upscias )

स्रोत: https://www.cba.am/Storage/EN/publications/statistics/monetary_stat_manual/sectors.pdf


38. भारत के सन्दर्भ में हाल ही में जनसंचार-माध्यमों में अक्सर चर्चित “अप्रत्यक्ष अंतरण” को निम्नलिखित में कौन-सी एक स्थिति सर्वोत्तम रूप से प्रतिबिंबित करती है?

  1. कोई भारतीय कंपनी, जिसने किसी विदेशी उद्यम में निवेश किया हो और अपने निवेश पर मिलने वाले लाभ पर उस बाहरी देश को कर अदा करती हो
  2. कोई विदेशी कंपनी, जिसने भारत में निवेश किया हो और अपने निवेश से मिलने वाले लाभ पर अपने आधारभूत देश को कर अदा करती हो
  3. कोई भारतीय कंपनी, जो किसी बाहरी देश में मूर्त संपत्ति खरीदती है और उनका मूल्य बढ़ने पर उन्हें बेच देती है तथा प्राप्ति को भारत में अंतरित कर देती है
  4. कोई विदेशी कंपनी शेयर अंतरित करती है और ऐसे शेयर भारत में स्थित परिसंपत्तियों से अपना वस्तुगत मूल्य व्युत्पन्न करते हैं।

Sol. Ans.(d). अप्रत्यक्ष अंतरण उन स्थितियों को संदर्भित करता है जहां विदेशी संस्थाओं के पास भारत में शेयर या संपत्ति होती है, और ऐसी विदेशी संस्थाओं के शेयरों को भारत में अंतर्निहित परिसंपत्तियों के प्रत्यक्ष अंतरण के बजाय स्थानांतरित किया जाता है। यह प्रश्न वोडाफोन शेयर अंतरण और पूर्वव्यापी कराधान विवाद के संदर्भ में प्रासंगिक था।

स्रोत:
https://kluwerlawonline.com/journalarticle/Intertax/45.10/TAXI2017057#:~:text=Indirect%20transfers%20refer%20to%20situations,the%20underlying%20assets%20in%20India


39. किसी संगठन या कंपनी द्वारा किए गए व्यय के सन्दर्भ में, निम्नलिखित कथनों में कौन-सा/से सही है/हैं?

  1. नई प्रौद्योगिकी प्राप्त करना पूँजीगत व्यय है।
  2. ऋण वित्तीयन को पूँजीगत व्यय माना जाता है, जबकि ईक्विटी वित्तीयन को राजस्व व्यय माना जाता है।

नीचे दिए कूट का प्रयोग कर सही उत्तर चुनिए:

  1. केवल 1
  2. केवल 2
  3. 1 और 2 दोनों
  4. न तो 1, न ही 2

Sol. Ans.(a). पूंजीगत व्यय वह धन है जो (सरकार या फर्मों द्वारा) मशीनरी, उपकरण, भवन, प्रौद्योगिकी, स्वास्थ्य सुविधाओं, शिक्षा, आदि के विकास पर खर्च किया जाता है। इसमें अचल संपत्तियों को प्राप्त करने पर किया गया खर्च भी शामिल है भूमि और निवेश जो भविष्य में लाभ या लाभांश देते हैं। आप ऋण वित्तपोषण द्वारा, या अन्य तरीकों से पूंजीगत व्यय कर सकते हैं।

राजस्व व्यय वह है जो न तो संपत्ति बनाता है और न ही सरकार के किसी दायित्व को कम करता है। कर्मचारियों का वेतन, पिछले कर्ज पर ब्याज भुगतान, सब्सिडी, पेंशन आदि । राजस्व व्यय की श्रेणी में आते हैं। यह प्रकृति में आवर्ती है। अतः इक्विटी वित्तीयन राजस्व व्यय नहीं हो सकता।

अतः, केवल 1 सही है।

स्रोत 1:
https://www.investopedia.com/terms/c/capitalexpenditure.asp#:~:text=Capital%20expenditures%20(CapEx)%20are%20funds,or%20investments%20by%20a%20company

स्रोत 2: https://www.business-standard.com/about/what-is-capital-expenditure


40. भारतीय अर्थव्यवस्था के सन्दर्भ में, निम्नलिखित कथनों पर विचार कीजिए:

  1. घरेलू वित्तीय बचत का एक भाग सरकारी ऋणग्रहण के लिए जाता है।
  2. नीलामी में बाज़ार-संबंधित दरों पर जारी दिनांकित प्रतिभूतियाँ, आंतरिक ऋण का एक बड़ा घटक होती हैं।

उपर्युक्त कथनों में कौन-सा/से सही है/हैं?

  1. केवल 1
  2. केवल 2
  3. 1 और 2 दोनों
  4. न तो 1, न ही 2

Sol. Ans.(c). सरकारी उधारी के लिए घरेलू बचत कितनी महत्वपूर्ण है? - घरेलू बचत के महत्व का अंदाजा इस बात से लगाया जा सकता है कि एक परिवार ही एकमात्र ऐसा खंड है जो अर्थव्यवस्था में शुद्ध वित्तीय अधिशेष क्षेत्र है; अन्य सभी खंड, अर्थात् सार्वजनिक क्षेत्र, निजी क्षेत्र और सामान्य सरकार, घाटे में हैं । साथ ही, मार्च 2019 के अंत में आंतरिक ऋण में विपणन योग्य प्रतिभूतियों की हिस्सेदारी 84.4 प्रतिशत थी, जो मार्च 2018 के अंत में 86.1 प्रतिशत से थोड़ा कम है

स्रोत 1 : https://www.financialexpress.com/opinion/how-critical-are-household-savings-for-government-borrowing/1542170/

स्रोत 2 : https://dea.gov.in/sites/default/files/Status%20Paper%20on%20Government%20Debt.pdf


41. निम्नलिखित फ़सलों में कौन-सी एक, मेथैन और नाइट्रस ऑक्साइड दोनों का सर्वाधिक महत्त्वपूर्ण मानवोद्भवी स्रोत है?

  1. कपास
  2. धान
  3. गन्ना
  4. गेहूँ

Sol. Ans.(b). बायोजेनिक मीथेन के महत्वपूर्ण मानवजनित स्रोत हैं गीले चावल के खेत, मवेशी, पशु अपशिष्ट, लैंडफिल और बायोमास जलाना। बायोजेनिक नाइट्रस ऑक्साइड के महत्वपूर्ण मानवजनित स्रोत भूमि-उपयोग परिवर्तन, उर्वरक उत्पादन और उपयोग और खाद का अनुप्रयोग हैं। पर्यावरण और पारिस्थितिकी और कृषि को मिलाकर एक उत्कृष्ट प्रश्न।

स्रोत 1: https://link.springer.com/article/10.1007/BF00748940


42. कृषि की “धान गहनता प्रणाली” का, जिसमें धान के खेतों का बारी-बारी से क्लेदन और शुष्कन किया जाता है, क्या परिणाम होता है?

  1. बीज की कम आवश्यकता
  2. मेथैन का कम उत्पादन
  3. बिजली की कम खपत

नीचे दिए कूट का प्रयोग कर सही उत्तर चुनिए:

  1. केवल 1 और 2
  2. केवल 2 और 3
  3. केवल 1 और 3
  4. 1, 2 और 3

Sol. Ans.(d). धान गहनता प्रणाली (एसआरआई) में जितना संभव हो उतना जैविक खाद के साथ चावल की खेती करना शामिल है। एसआरआई धान की खेती में बीज की कम मात्रा - 2 किग्रा/एकड़ की आवश्यकता होती है। अनुसंधान से पता चला है कि एसआरआई या एडब्ल्यूडी द्वारा धान की रुक-रुक कर सिंचाई करने से मीथेन उत्सर्जन में 22% से 64% के बीच कमी आई है।

एसआरआई के तहत, प्रक्रिया एक वर्ग पैटर्न में व्यापक दूरी पर अकेले लगाए गए युवा रोपणों के साथ टार्ट करती है; और आंतरायिक सिंचाई के साथ जो मिट्टी को नम रखता है लेकिन जलमग्न नहीं होता है, और सक्रिय रूप से वीडर के साथ लगातार अंतर खेती करता है। चूंकि कम सिंचाई की आवश्यकता होती है, इससे बिजली की खपत कम हो जाती है। अत: तीनों कथन सही हैं।

स्रोत 1: https://vikaspedia.in/agriculture/best-practices/sustainable-agriculture/crop-management/sri-2013-new-method-of-growing-rice

स्रोत 2: http://sri.ciifad.cornell.edu/index_files/ClimateChangeMitigation.pdf


43. पश्चिम अफ्रीका की निम्नलिखित झीलों में कौन-सी एक, सूख कर मरुस्थल में बदल गई है?

  1. लेक विक्टोरिया
  2. लेक फागुबिन
  3. लेक ओगुटा
  4. लेक वोल्टा

Sol. Ans.(b). विक्टोरिया झील पूर्वी अफ्रीका (तंजानिया, युगांडा और केन्या की सीमा पर) में है।

फगुइबिन झील, टिम्बकटू से 80 किमी पश्चिम में और नाइजर नदी से 75 किमी उत्तर में स्थित सहारा रेगिस्तान के दक्षिणी किनारे पर माली में एक झील है, जिससे यह छोटी झीलों और चैनलों की एक प्रणाली से जुड़ी हुई है। वर्षों में जब नदी की वार्षिक बाढ़ की ऊंचाई पर्याप्त होती है, नदी से पानी झील में बहता है। 1970 और 1980 के दशक के साहेल सूखे के बाद से झील ज्यादातर सूखी रही है। झील में पानी बहुत कम ही पहुंचा है और जब ऐसा हुआ भी है अतः झील में आंशिक रूप से ही पानी भरा हुआ है। इससे स्थानीय पारिस्थितिकी तंत्र का आंशिक पतन हुआ है।


44. दक्षिण भारत की गंडिकोटा घाटी (कैन्यन) निम्नलिखित नदियों में से किस एक से निर्मित हुई है

  1. कावेरी
  2. मंजिरा
  3. पेन्नार
  4. तुंगभद्रा

Sol. Ans.(c). गंडिकोटा आंध्र प्रदेश के कडपा जिले का एक छोटा सा गाँव है जो पेन्नार नदी द्वारा बनाई गई अपनी शानदार घाटी के लिए जाना जाता है जो एर्रामला पहाड़ियों से होकर गुजरती है। प्रकृति की वास्तुकला के इस सुरम्य टुकड़े को भारत के हिडन ग्रैंड कैन्यन के रूप में जाना जाता है , जो इसे देखने और इसकी सुंदरता से मंत्रमुग्ध हो जाते हैं। भूगोल पर एक बहुत ही विशिष्ट प्रश्न।


45. निम्नलिखित युग्मों पर विचार कीजिए:

उपर्युक्त युग्मों में कौन-सा/से सही सुमेलित है/हैं?

  1. 1 और 2
  2. केवल 2
  3. 1 और 3
  4. केवल 3

Sol. Ans.(b). नमचा बरवा पूर्वी हिमालय - अरुणाचल प्रदेश में स्थित है। गढ़वाल हिमालय उत्तराखंड में है। नंदा देवी उत्तराखंड राज्य में विशेष रूप से कुमाऊं हिमालय में स्थित है। नोकरेक मेघालय राज्य में गारो हिल्स में स्थित है। अतः केवल 2 ही सही है, 1 और 3 नहीं

( यह प्रश्न सीधे पीटी के पावरपैक कोर्स में शामिल था - यहां नामांकन करें -https://bit.ly/upscias )


46. अक्सर समाचारों में सुनाई देने वाला शब्द "लिट”मोटे तौर पर निम्नलिखित में से किस क्षेत्र से संगत है?

  1. पूर्वी भूमध्यसागरीय तट के पास का क्षेत्र
  2. उत्तरी अफ्रीकी तट के पास का मिस्र से मोरक्को तक फैला क्षेत्र
  3. फारस की खाड़ी और अफ्रीका के शृंग (हॉर्न - ऑफ़ अफ्रीका) के पास का क्षेत्र
  4. भूमध्य सागर के सम्पूर्ण तटवर्ती क्षेत्र

Sol. Ans.(a). लेवेंट (Levant) एक अनुमानित ऐतिहासिक भौगोलिक शब्द है जो पश्चिमी एशिया के पूर्वी भूमध्य क्षेत्र में एक बड़े क्षेत्र का जिक्र करता है।

( यह प्रश्न सीधे पीटी के पावरपैक कोर्स में शामिल था - यहां नामांकन करें -https://bit.ly/upscias )


47. निम्नलिखित देशों पर विचार कीजिए:

  1. अज़रबैजान
  2. किरगिज़स्तान
  3. ताजिकिस्तान
  4. तुर्कमेनिस्तान
  5. उज़्बेकिस्तान

उपर्युक्त में से किनकी सीमाएँ अफगानिस्तान के साथ लगती हैं?

  1. केवल 1, 2 और 5
  2. केवल 1, 2, 3 और 4
  3. केवल 3, 4 और 5
  4. 1, 2, 3, 4 और 5

Sol. Ans.(c). दिए गए विकल्पों में से अफगानिस्तान के सीमावर्ती देश तुर्कमेनिस्तान, उज्बेकिस्तान और ताजिकिस्तान हैं। ध्यान दें कि किर्गिस्तान इसकी सीमा नहीं है। यह सीधे तीन विकल्प (ए, बी और डी) को समाप्त करता है। वास्तव में एक सितार!

( यह प्रश्न सीधे पीटी के पावरपैक कोर्स में शामिल था - यहां नामांकन करें -https://bit.ly/upscias )


48. भारत के सन्दर्भ में, निम्नलिखित कथनों पर विचार कीजिए:

  1. मोनाज़ाइट दुर्लभ मृदाओं का स्रोत है ।
  2. मोनाज़ाइट में थोरियम होता है ।
  3. भारत की समस्त तटवर्ती बालुकाओं में मोनाज़ाइट प्राकृतिक रूप में होता है।
  4. भारत में, केवल सरकारी निकाय ही मोनाज़ाइट ____संसाधित या निर्यात कर सकते हैं ।

उपर्युक्त कथनों में कौन-से सही हैं?

  1. केवल 1, 2 और 3
  2. केवल 1, 2 और 4
  3. केवल 3 और 4
  4. 1, 2, 3 और 4

Sol. Ans.(b). मोनाज़ाइट मुख्य रूप से लाल-भूरे रंग का फॉस्फेट खनिज है जिसमें दुर्लभ-पृथ्वी तत्व होते हैं। मोनाजाइट की थोरियम सामग्री परिवर्तनशील है और कभी-कभी 20-30% तक हो सकती है। कुछ कार्बोनेट या बोलिवियाई टिन अयस्क शिराओं से प्राप्त मोनाज़ाइट अनिवार्य रूप से थोरियम मुक्त होता है। हालांकि, वाणिज्यिक मोनाजाइट रेत में आमतौर पर 6 से 12% थोरियम ऑक्साइड होता है। मोनाज़ाइट भारत में पूरे पूर्वी तट और केरल तटीय रेत में प्राकृतिक रूप से पाया जाता है (जिसे संपूर्ण पश्चिमी समुद्र तट नहीं माना जा सकता)। अतः 1 और 2 सत्य हैं परन्तु 3 नहीं है। यदि 3 गलत है, तो तीन विकल्पों को तुरंत खारिज कर दिया जाता है (ए, सी और डी)। इसके अलावा, केवल राज्य द्वारा संचालित इंडियन रेयर अर्थ्स लिमिटेड (IREL) थोरियम युक्त समुद्र तट रेत खनिज मोनाजाइट का उत्पादन और प्रसंस्करण कर सकता है। (यह प्रश्न पीटी के पावरपैक कोर्स में शामिल था -

( यह प्रश्न सीधे पीटी के पावरपैक कोर्स में शामिल था - यहां नामांकन करें -https://bit.ly/upscias )

स्रोत 1:
https://economictimes.indiatimes.com/industry/indl-goods/svs/metals-mining/no-private-licence-given-for-monazite-production-dae/articleshow/16900031.cms?utm_source=contentofinterest&utm_medium=text&utm_campaign=cppst

स्रोत 2: https://en.wikipedia.org/wiki/Monazite


49. उत्तरी गोलार्ध में, वर्ष का सबसे लंबा दिन आम तौर पर कब होता है?

  1. जून महीने का पहला पखवाड़ा
  2. जून महीने का दूसरा पखवाड़ा
  3. जुलाई महीने का पहला पखवाड़ा
  4. जुलाई महीने का दूसरा पखवाड़ा

Sol. Ans.(c). भूमध्य रेखा के उत्तर में रहने वालों के लिए सबसे लंबा दिन 21 जून है । तकनीकी शब्दों में, इस दिन को ग्रीष्म संक्रांति कहा जाता है , जो ग्रीष्म ऋतु का सबसे लंबा दिन होता है। यह तब होता है जब सूर्य सीधे कर्क रेखा पर होता है। भूगोल से एक मानक प्रश्न।

( यह प्रश्न सीधे पीटी के पावरपैक कोर्स में शामिल था - यहां नामांकन करें -https://bit.ly/upscias )


50. निम्नलिखित युग्मों पर विचार कीजिए:

उपर्युक्त युग्मों में कितने सही सुमेलित हैं?

  1. केवल एक युग्म
  2. केवल दो युग्म
  3. केवल तीन युग्म
  4. सभी चारों युग्म

Sol. Ans.(b). होकेरा वेटलैंड - जम्मू और कश्मीर, रेणुका वेटलैंड - हिमाचल प्रदेश, रुद्रसागर झील - त्रिपुरा, सस्थामकोट्टा झील - केरल। केवल दो जोड़े सही हैं। अब इस प्रश्न के लिए आपको वास्तव में भारत में सभी आर्द्रभूमियों और उनके स्थानों को याद रखना होगा!

( यह प्रश्न सीधे पीटी के पावरपैक कोर्स में शामिल था - यहां नामांकन करें -https://bit.ly/upscias )



51. निम्नलिखित युग्मों पर विचार कीजिए:

उपर्युक्त युग्मों में से कितने सही सुमेलित हैं?

  1. केवल एक युग्म
  2. केवल दो युग्म
  3. केवल तीन युग्म
  4. सभी चारों युग्म

Sol. Ans.(b). यह "प्राचीन और मध्यकालीन इतिहास" से एक प्रश्न है। धौली ओडिशा में स्थित है। एरागुडी या येरागुडी आंध्र प्रदेश में स्थित है। जौगड़ा ओडिशा में स्थित है। कालसी उत्तराखंड में स्थित है। केवल दो जोड़े सही हैं। एक कठिन प्रश्न क्योंकि इसमें आपको सम्राट अशोक के सभी शिलालेख याद रखने होंगे।

(यह प्रश्न सीधे पीटी के पावरपैक कोर्स में शामिल था - यहां नामांकन करें -https://bit.ly/upscias )

स्रोत 1: http://www.columbia.edu/itc/ealac/landesman/summer_public_html/week1/maps/ashokamap.html

स्रोत 2: https://www.worldhistory.org/Edicts_of_Ashoka/


52. निम्नलिखित युग्मों पर विचार कीजिए :

उपर्युक्त युग्मों में कितने सही सुमेलित हैं?

  1. केवल एक युग्म
  2. केवल दो युग्म
  3. केवल तीन युग्म
  4. सभी चारों युग्म

Sol. Ans.(b). नन्नूका (831−845 ईस्वी) चंदेलों के संस्थापक राजा थे।

जयशक्ति और विजयशक्ति (865-885 ईस्वी) वाक्पति के पुत्रों जयशक्ति (जेजा) और विजयशक्ति (विजा) ने चंदेल शक्ति को समेकित किया। महोबा शिलालेख के अनुसार जयशक्ति के नाम पर चंदेल क्षेत्र का नाम "जेजाकभुक्ति" रखा गया। अतः, जयशक्ति भी चंदेल वंश के थे।

नागभट्ट द्वितीय (800-833 ईस्वी): नागभट्ट द्वितीय को उनके पुत्र रामभद्र ने उत्तराधिकारी बनाया, जिन्होंने कुछ समय तक शासन किया, और उनके पुत्र मिहिरा भोज ने उनका उत्तराधिकारी बना लिया। नागभट्ट द्वितीय प्रतिहार वंश का है।

भोज प्रथम / मिहिर भोज (836−885 ईस्वी): नागभट्ट द्वितीय का पोता, जिसने 46 वर्षों से अधिक का लंबा शासन किया और प्रतिहारों का सबसे सफल और लोकप्रिय शासक साबित हुआ। अतः युग्म 2 और 4 सही नहीं हैं।

आपसे भारत के प्रमुख ऐतिहासिक राजवंशों के सभी प्रमुख सम्राटों के नाम याद रखने की अपेक्षा की गई थी। एक कठिन प्रश्न।

( यह प्रश्न सीधे पीटी के पावरपैक कोर्स में शामिल था - यहां नामांकन करें -https://bit.ly/upscias )


53. प्राचीन दक्षिण भारत में संगम साहित्य के बारे में, निम्नलिखित कथनों में कौन-सा एक, सही है?

  1. संगम कविताओं में भौतिक संस्कृति का कोई सन्दर्भ नहीं है।
  2. वर्ण का सामाजिक वर्गीकरण संगम कवियों को ज्ञात था।
  3. संगम कविताओं में समर शौर्य का कोई सन्दर्भ नहीं है।
  4. संगम साहित्य में जादुई ताकतों को असंगत बताया गया है।

Sol. Ans.(b). संगम युग के कवियों को वर्ण का सामाजिक वर्गीकरण ज्ञात था। इसमें अरशर (राजा), वैश्यियर (व्यापारी) और वेलालर (किसान) का उल्लेख है। ब्राह्मणों का भी उल्लेख है। हालाँकि, "4 गुना वर्ण वर्गीकरण" का प्राचीन तमिल समाज में बहुत कम अनुप्रयोग था। वर्गीकरण का अधिक प्रासंगिक आधार कुटी था जो वंश-आधारित वंश समूह थे। हालांकि वंश और वंशानुगत व्यवसायों से जुड़े, कुटी समूहों के बीच अंतर-भोजन और सामाजिक बातचीत पर कोई वास्तविक प्रतिबंध नहीं था।


54. किसके राज्यकाल में “योगवाशिष्ठ”का निज़ामुद्दीन पानीपति द्वारा फ़ारसी में अनुवाद किया गया?

  1. अकबर
  2. हुमायु
  3. शाहजहाँ
  4. औरंगजेब

Sol. Ans.(a). मुगल राजवंश के दौरान अकबर, जहांगीर और दारा शिकोह के आदेश के अनुसार पाठ का कई बार फारसी में अनुवाद किया गया था। इनमें से एक अनुवाद निजाम अल-दीन पानीपति ने सोलहवीं शताब्दी के अंत में किया था। एक बहुत ही सटीक प्रश्न, विशिष्ट ज्ञान की आवश्यकता है।


55. हाल ही में हैदराबाद में भारत के प्रधान मंत्री द्वारा रामानुज की आसन मुद्रा में विश्व की दूसरी सबसे ऊँची मूर्ति का उद्घाटन किया गया था। निम्नलिखित कथनों में कौन-सा एक, रामानुज की शिक्षाओं को सही निरूपित करता है?

  1. मोक्ष प्राप्ति का सर्वोत्तम साधन भक्ति था ।
  2. वेद शाश्वत, आत्म-प्रतिष्ठित तथा पूर्णतया प्रामाणिक हैं।
  3. तर्कसंगत युक्तियाँ सर्वोच्च आनंद के मौलिक माध्यम थे।
  4. ध्यान के माध्यम से मोक्ष पाया जा सकता था ।

Sol. Ans.(a). रामानुजाचार्य एक हिंदू धर्मशास्त्री, दार्शनिक और हिंदू धर्म के भीतर श्री वैष्णववाद परंपरा के सबसे महत्वपूर्ण समर्थक थे। रामानुजाचार्य के दर्शन को विशिष्टद्वैत या योग्य अद्वैतवाद के रूप में जाना जाने लगा । उनके अनुसार मोक्ष प्राप्ति के लिए भक्ति और समर्पण महत्वपूर्ण है। इस प्रश्न को हल करने का एक आसान तरीका यह याद रखना था कि रामानुज भक्ति युग के संत थे। यदि ऐसा है, तो विकल्प (b), (c) और (d) खारिज किये जा सकते है।

स्रोत: https://www.britannica.com/biography/Ramanuja


56. हाल ही में, प्रधान मंत्री ने वेरावल में सोमनाथ मंदिर के निकट नए सर्किट हाउस का उद्घाटन किया । सोमनाथ मंदिर के बारे में निम्नलिखित कथनों में कौन-से सही है?

  1. सोमनाथ मंदिर ज्योतिर्लिंग देव-मंदिरों में से एक है ।
  2. अल-बरूनी ने सोमनाथ मंदिर का वर्णन किया है।
  3. सोमनाथ मंदिर की प्राण-प्रतिष्ठा (आज के मंदिर की स्थापना) राष्ट्रपति एस. राधाकृष्णन द्वारा की गई थी।

नीचे दिए कूट का प्रयोग कर सही उत्तर चुनिए:

  1. केवल 1 और 2
  2. केवल 2 और 3
  3. केवल 1 और 3
  4. 1, 2 और 3

Sol. Ans.(a). एक ज्योतिर्लिंग या ज्योतिर्लिंगम, हिंदू भगवान शिव का एक भक्ति प्रतिनिधित्व है। यह शब्द ज्योति 'चमक' और लिंग का संस्कृत यौगिक है। शिव पुराण में भारत में 64 मूल ज्योतिर्लिंग मंदिरों का उल्लेख है, जिनमें से 12 सबसे पवित्र (महा ज्योतिर्लिंगम) (महान ज्योतिर्लिंग) हैं। सोमनाथ उनमें से एक हैं। अतः, 1 सही है।

मध्ययुगीन युग के पोलीमैथ अल बरूनी ने मंदिर के विनाश का वर्णन किया: "जनवरी 1026 में, कई भक्तों को मारने के बाद, सोमनाथ लिंगम को तोड़ा गया, और लूट की राशि एक बड़ी राशि थी"। अत: कथन 2 भी सही है।

सोमनाथ मंदिर की प्राण-प्रतिष्ठा (स्थापना) भारत के तत्कालीन राष्ट्रपति डॉ. राजेंद्र प्रसाद द्वारा की गई थी। अतः, 3 सही नहीं है।

स्रोत : https://en.wikipedia.org/wiki/Jyotirlinga


57. निम्नलिखित कथनों में कौन-सा एक, मानव शरीर में B कोशिकाओं और T कोशिकाओं की भूमिका का सर्वोत्तम वर्णन है?

  1. वे शरीर को पर्यावरणीय प्रत्यूर्जकों (एलर्जनों) से संरक्षित करती हैं।
  2. वे शरीर के दर्द और सूजन का अपशमन करती हैं ।
  3. वे शरीर में प्रतिरक्षा-निरोधकों की तरह काम करती
  4. वे शरीर को रोगजनकों द्वारा होने वाले रोगों से बचाती हैं।

Sol. Ans.(d). B कोशिकाएं एंटीबॉडी बनाती हैं। B लिम्फोसाइट्स, जिन्हें B कोशिकाएं भी कहा जाता है, एक प्रकार का प्रोटीन बनाते हैं जिसे एंटीबॉडी कहा जाता है। T कोशिकाएं संक्रमित या कैंसर कोशिकाओं का सफाया कर सकती हैं। वे हमलावर रोगजनकों को खत्म करने के लिए B लिम्फोसाइटों की मदद करके प्रतिरक्षा प्रतिक्रिया को भी निर्देशित करते हैं। यह जीव विज्ञान (सामान्य विज्ञान), या विज्ञान और प्रौद्योगिकी से एक प्रश्न है।

स्रोत:
https://www.mdanderson.org/cancerwise/t-cells--b-cells-and-the-immune-system.h00-159465579.html#:~:text=They%20also%20direct%20the%20immune,as%20toxins%2C%20to%20neutralize%20them


58. निम्नलिखित कथनों पर विचार कीजिए:

  1. परासूक्ष्मकण (नैनोपार्टिकल्स), मानव-निर्मित होने के सिवाय, प्रकृति में अस्तित्व में नहीं हैं।
  2. कुछ धात्विक ऑक्साइडों के परासूक्ष्मकण, प्रसाधन-सामग्री (कॉस्मेटिक्स) के निर्माण में काम आते हैं।
  3. कुछ वाणिज्यिक उत्पादों के परासूक्ष्मकण, जो पर्यावरण में आ जाते हैं, मनुष्यों के लिए असुरक्षित हैं।

उपर्युक्त कथनों में कौन-सा/से सही है/हैं?

  1. केवल 1
  2. केवल 3
  3. 1 और 2
  4. 2 और 3

Sol. Ans.(d). कथन 1 सही नहीं है। प्राकृतिक रूप से पाए जाने वाले परासूक्ष्मकण ज्वालामुखी की राख, समुद्री स्प्रे, महीन रेत और धूल, और यहां तक कि जैविक पदार्थ (जैसे वायरस) में भी पाए जा सकता है। अतः, (a) और (c) गलत हैं। अब, कथन 2 सही है। कॉस्मेटिक उद्योग में अकार्बनिक नैनोकणों के अनुप्रयोगों में होंठ, नाखून, बाल और त्वचा की देखभाल सहित विविध डोमेन शामिल हैं, और पिछले कुछ दशकों में इसमें वृद्धि देखी गई है। अतः केवल विकल्प (d) संभव है। (यह प्रश्न पीटी के पावरपैक कोर्स में शामिल था

( यह प्रश्न सीधे पीटी के पावरपैक कोर्स में शामिल था - यहां नामांकन करें -https://bit.ly/upscias )

स्रोत:
https://www.ncbi.nlm.nih.gov/pmc/articles/PMC8417693/

स्रोत:
https://sustainable-nano.com/2013/03/25/nanoparticles-are-all-around-us/#:~:text=Naturally%20occurring%20nanoparticles%20can%20be,than%20their%20naturally%20occurring%20counterparts


59. निम्नलिखित कथनों पर विचार कीजिए:

DNA बारकोडिंग किसका उपसाधन हो सकता है?

  1. किसी पादप या प्राणी की आयु का आकलन करने के लिए
  2. समान दिखने वाली प्रजातियों के बीच भिन्नता - जानने के लिए
  3. प्रसंस्कृत खाद्यपदार्थों में अवांछित प्राणी या पादप सामग्री को पहचानने के लिए।

उपर्युक्त कथनों में कौन-सा/से सही है/हैं?

  1. केवल 1
  2. केवल 3
  3. 1 और 2
  4. 2 और 3

Sol. Ans.(d). डीएनए बारकोडिंग एक विशिष्ट जीन या जीन से डीएनए के एक छोटे खंड का उपयोग करके प्रजातियों की पहचान की एक विधि है। डीएनए बारकोडिंग एक व्यापक रूप से उपयोग की जाने वाली आणविक-आधारित प्रणाली है, जो जैविक नमूनों की पहचान कर सकती है, और इसका उपयोग कच्चे माल और प्रसंस्कृत भोजन दोनों की पहचान के लिए किया जाता है। अतः, 2 और 3 सही हैं, और केवल संभावित विकल्प (d) है।

स्रोत 1: http://citeseerx.ist.psu.edu/viewdoc/download?doi=10.1.1.717.1235&rep=rep1&type=pdf

स्रोत: 2 : https://en.wikipedia.org/wiki/DNA_barcoding


60. निम्नलिखित पर विचार कीजिए:

  1. कार्बन मोनोक्साइड
  2. नाइट्रोजन ऑक्साइड
  3. ओज़ोन
  4. सल्फर डाइऑक्साइड

वातावरण में उपर्युक्त में से किसकी/किनकी अधिकता होने से अम्ल वर्षा होती है?

  1. 1, 2 और 3
  2. केवल 2 और 4
  3. केवल 4
  4. 1, 3 और 4

Sol. Ans.(b). अम्लीय वर्षा एक रासायनिक प्रतिक्रिया के कारण होती है जो तब शुरू होती है जब सल्फर डाइऑक्साइड और नाइट्रोजन ऑक्साइड जैसे यौगिक हवा में उत्सर्जित किये जाते हैं। ये पदार्थ वातावरण में बहुत अधिक बढ़ सकते हैं, जहां वे पानी, ऑक्सीजन और अन्य रसायनों के साथ मिश्रित और प्रतिक्रिया करते हैं और अधिक अम्लीय प्रदूषक बनाते हैं, जिसे अम्लीय वर्षा (एसिड रेन) के रूप में जाना जाता है। अतः, केवल 2 और 4 सही हैं।


61. निम्नलिखित कथनों पर विचार कीजिए:

  1. उच्च मेघ मुख्यत: सौर विकिरण को परावर्तित कर भूपृष्ठ को ठंडा करते हैं ।
  2. भूपृष्ठ से उत्सर्जित होने वाली अवरक्त विकिरणों का निम्न मेघ में उच्च अवशोषण होता है, और इससे तापन प्रभाव होता है।

उपर्युक्त कथनों में कौन-सा/से सही है/हैं?

  1. केवल 1
  2. केवल 2
  3. 1 और 2 दोनों
  4. न तो 1, न ही 2

Sol. Ans.(d). दोनों कथन गलत हैं, और उनका उल्टा सही होगा।

उच्च, पतले बादल मुख्य रूप से आने वाले सौर विकिरण को संचारित करते हैं; साथ ही, वे पृथ्वी द्वारा उत्सर्जित कुछ निवर्तमान अवरक्त विकिरणों में फंस जाते हैं और इसे वापस नीचे की ओर विकीर्ण कर देते हैं, जिससे पृथ्वी की सतह गर्म हो जाती है। कम, घने बादल मुख्य रूप से सौर विकिरण को दर्शाते हैं और पृथ्वी की सतह को ठंडा करते हैं। अत: दोनों कथन सही नहीं हैं।

स्रोत: https://earthobservatory.nasa.gov/features/Clouds#:~:text=Low%2C%20thick%20clouds%20primarily%20reflect,the%20surface%20of%20the%20Earth


62. निम्नलिखित कथनों पर विचार कीजिए:

  1. उत्तरी-पश्चिमी केन्या में बीड़ीबीड़ी एक वृहद् शरणार्थी बस्ती है।
  2. दक्षिण सूडान गृह युद्ध से पलायन किए हुए कुछ लोग बीड़ीबीड़ी में रहते हैं।
  3. सोमालिया के गृह युद्ध से पलायन किए हुए कुछ लोग केन्या के ददाब शरणार्थी संकुल में रहते हैं ।

उपर्युक्त कथनों में कौन-सा/से सही है/हैं?

  1. 1 और 2
  2. केवल 2
  3. 2 और 3
  4. केवल 3

Sol. Ans.(c). बीड़ीबीड़ी शरणार्थी बस्ती उत्तर पश्चिमी युगांडा में एक शरणार्थी शिविर है (और केन्या नहीं, जैसा कि प्रश्न में दिया गया है)। 270,000 से अधिक दक्षिण सूडानी शरणार्थी चल रहे गृहयुद्ध से भाग रहे हैं, 2017 की शुरुआत में यह दुनिया का सबसे बड़ा शरणार्थी शरण स्थल था। अतः 1 सही नहीं है और 2 सही है।

ददाब शरणार्थी परिसर में जुलाई 2020 के अंत तक 2,18,873 पंजीकृत शरणार्थियों और शरण चाहने वालों की आबादी है। ददाब शरणार्थी परिसर में तीन शिविर हैं। पहला शिविर 1991 में स्थापित किया गया था, जब सोमालिया में गृहयुद्ध से भागे शरणार्थियों ने केन्या में सीमा पार करना शुरू किया। 2011 में दूसरी बड़ी बाढ़ आई, जब दक्षिणी सोमालिया में सूखे और अकाल से भागकर करीब 1,30,000 शरणार्थी पहुंचे ।


63. निम्नलिखित देशों पर विचार कीजिए:

  1. आर्मीनिया
  2. अज़रबैजान
  3. क्रोएशिया
  4. रोमानिया
  5. उज़्बेकिस्तान

उपर्युक्त में कौन-से तुर्की राज्यों के संगठन के सदस्य हैं?

  1. 1, 2 और 4
  2. 1 और 3
  3. 2 और 5
  4. 3, 4 और 5

Sol. Ans.(c). तुर्की राज्यों का संगठन, जिसे पहले तुर्की परिषद या तुर्की भाषी राज्यों की सहयोग परिषद कहा जाता था, एक अंतरराष्ट्रीय संगठन है जिसमें प्रमुख स्वतंत्र तुर्क देश शामिल हैं: अज़रबैजान, कजाकिस्तान, किर्गिस्तान, तुर्की और उजबेकिस्तान। दिए गए देशों में से हमारा 2 (अजरबैजान) और 5 (उजबेकिस्तान) सही है। (यदि आप सभी सदस्यों के नाम याद नहीं कर सकते हैं, अतः इस विशेष प्रश्न में, यह सोचने का प्रयास करें कि 11 वीं शताब्दी से भारत में आने वाले तुर्क समूहों का हिस्सा कौन थे - स्पष्ट रूप से, 1, 2 और 4 नहीं थे)।


64. निम्नलिखित कथनों पर विचार कीजिए:

  1. गुजरात में भारत का विशालतम सौर पार्क है ।
  2. केरल में पूर्णत: सौर शक्तिकृत अंतर्राष्ट्रीय हवाई अड्डा है।
  3. गोआ में भारत की विशालतम तैरती हुई सौर प्रकाश-वोल्टीय परियोजना है।

उपर्युक्त कथनों में कौन-सा/से सही है/हैं?

  1. 1 और 2
  2. केवल 2
  3. 1 और 3
  4. केवल 3

Sol. Ans.(b). भादला सौर पार्क, जोधपुर, राजस्थान न केवल भारत बल्कि दुनिया में सबसे बड़ा सौर ऊर्जा पार्क है। यह 14,000 एकड़ में फैला है। पार्क में 10 मिलियन से अधिक सौर पैनल हैं, जो 2245MW की परिचालन क्षमता में योगदान करते हैं। अतः, 1 सही नहीं है। अतः (a) और (c) खारिज कर दिया जाता है। फर्म भारत हेवी इलेक्ट्रिकल्स लिमिटेड (भेल) ने भारत के सबसे बड़े फ्लोटिंग सोलर पीवी प्लांट को सफलतापूर्वक चालू कर दिया है। आंध्र प्रदेश में एनटीपीसी सिम्हाद्री में स्थित, 25 मेगावाट की फ्लोटिंग एसपीवी परियोजना 100 एकड़ के क्षेत्र में फैली हुई है। अतः, 3 सही नहीं है। कोचीन इंटरनेशनल एयरपोर्ट लिमिटेड (CIAL) पूरी तरह से सौर ऊर्जा से संचालित दुनिया के पहले हवाई अड्डे का मालिक है।

केवल संभव विकल्प (b) है।

स्रोत: https://www.thehindu.com/sci-tech/energy-and-environment/worlds-largest-solar-park-in-bhadla-india/article37462665.ece

स्रोत:
https://www.livemint.com/industry/energy/indias-largest-floating-solar-power-plant-commissioned-in-andhra-pradesh-11631774123853.html

स्रोत: https://www.indiatimes.com/news/india/kochi-airport-more-solar-power-with-floating-solar-plant-532356.html


65. समुद्री कानून पर संयुक्त राष्ट्र अभिसमय (युनाइटेड नेशंस कन्वेंशन) के संदर्भ में, निम्नलिखित कथनों पर विचार कीजिए:

  1. किसी तटीय राज्य को, अपने प्रादेशिक समुद्र की चौड़ाई को, आधार-रेखा से मापित, 12 समुद्री मील से अनधिक सीमा तक अभिसमय के अनुरूप सुस्थापित करने का अधिकार है।
  2. सभी राज्यों के, चाहे वे तटीय हों या भू-बद्ध भाग के हों, जहाजों को प्रादेशिक समुद्र से हो कर बिना रोक-टोक यात्रा का अधिकार होता है ।
  3. अनन्य आर्थिक क्षेत्र का विस्तार उस आधार-रेखा से 200 समद्री मील से अधिक नहीं होगा, जहाँ से प्रादेशिक समुद्र की चौड़ाई मापी जाती है ।

उपर्युक्त कथनों में कौन-से सही हैं?

  1. केवल 1 और 2
  2. केवल 2 और 3
  3. केवल 1 और 3
  4. 1, 2 और 3

Sol. Ans.(d). प्रत्येक राज्य को अपने प्रादेशिक समुद्र की चौड़ाई को 12 समुद्री मील से अधिक की सीमा तक स्थापित करने का अधिकार है, जिसे इस कन्वेंशन के अनुसार निर्धारित बेसलाइन से मापा जाता है। अतः, 1 सही है। इस कन्वेंशन के अधीन, सभी राज्यों के जहाज, चाहे वे तटीय हों या भूआवेष्टित, प्रादेशिक समुद्र से बिना रोक-टोक यात्रा का अधिकार होता हैं। अतः, 2 भी सही है। एक "अनन्य आर्थिक क्षेत्र" या "ईईजेड" महासागर का एक क्षेत्र है, जो आम तौर पर किसी देश के क्षेत्रीय समुद्र से 200 समुद्री मील (230 मील) तक फैला हुआ है, जिसके भीतर एक तटीय राष्ट्र के पास जीवित और निर्जीव दोनों संसाधनों पर अधिकार क्षेत्र है। अतः, 3 भी सही है।

( यह प्रश्न सीधे पीटी के पावरपैक कोर्स में शामिल था - यहां नामांकन करें -https://bit.ly/upscias )

स्रोत: https://www.un.org/depts/los/convention_agreements/texts/unclos/part2.htm


66. निम्नलिखित कथनों में कौन-सा एक, कभी-कभी समाचारों में उल्लिखित सेंकाकू द्वीप विवाद को सर्वोत्तम रूप में प्रतिबिंबित करता है?

  1. आम तौर पर यह माना जाता है कि वे दक्षिणी चीन सागर के आसपास किसी देश द्वारा निर्मित कृत्रिम द्वीप हैं।
  2. चीन और जापान के बीच पूर्वी चीन सागर में इन द्वीपों के विषय में समुद्री विवाद होता रहता है।
  3. वहाँ ताइवान को अपनी रक्षा क्षमताओं को बढ़ाने में मदद करने के लिए एक स्थायी अमेरिकी सैन्य अड्डा स्थापित किया गया है।
  4. यद्यपि अंतर्राष्ट्रीय न्यायालय ने उन्हें अस्वामिक भूमि घोषित किया है, तथापि कुछ दक्षिण-पूर्वी एशियाई देश उन पर दावा करते हैं ।

Sol. Ans.(b). सेनकाकू द्वीप विवाद, या डियाओयू द्वीप विवाद, निर्जन द्वीपों के एक समूह पर एक क्षेत्रीय विवाद है जिसे जापान में सेनकाकू द्वीप समूह, पीपुल्स रिपब्लिक ऑफ चाइना (पीआरसी) में दियाओयू द्वीप समूह और चीन गणराज्य (आरओसी या ताइवान) में टियाओयुताई द्वीप समूह के रूप में जाना जाता है। । यह काफी पुराना सवाल है जो कई सालों बाद पूछा जा रहा है।

( यह प्रश्न सीधे पीटी के पावरपैक कोर्स में शामिल था - यहां नामांकन करें -https://bit.ly/upscias )


67. निम्नलिखित युग्मों पर विचार कीजिए:

उपर्युक्त युग्मों में कितने सही सुमेलित हैं?

  1. केवल एक युग्म
  2. केवल दो युग्म
  3. केवल तीन युग्म
  4. सभी चारों युग्म

Sol. Ans.(c). जिबूती में चीनी पीपुल्स लिबरेशन आर्मी सपोर्ट बेस, अफ्रीका के हॉर्न में जिबूती में स्थित चीनी पीपुल्स लिबरेशन आर्मी नेवी (पीएलएएन) द्वारा संचालित एक सैन्य अड्डा है। अन्य तीन सही सुमेलित हैं। इस तरह के प्रश्नों में, आपको पूछे गए सभी चार चीजों के बारे में जानना होगा अन्यथा प्रश्न को ठीक से हल नहीं किया जा सकता है।


68. निम्नलिखित युग्मों पर विचार कीजिए:

उपर्युक्त युग्मों में कितने सही सुमेलित हैं?

  1. केवल एक युग्म
  2. केवल दो युग्म
  3. केवल तीन युग्म
  4. सभी चारों युग्म

Sol. Ans.(b). अनातोलिया , जिसे एशिया माइनर के नाम से भी जाना जाता है, पश्चिमी एशिया में एक बड़ा प्रायद्वीप और एशियाई महाद्वीप का पश्चिमीतम फलाव है । यह आधुनिक तुर्की के प्रमुख भाग का गठन करता है।

अम्हारा एक सेमिटिक-भाषी जातीय समूह है जो इथियोपिया के स्वदेशी है, पारंपरिक रूप से इथियोपिया के उत्तर-पश्चिमी हाइलैंड्स के कुछ हिस्सों में निवास करता है, विशेष रूप से अम्हारा क्षेत्र में निवास करता है।

काबो डेलगाडो मोजाम्बिक का सबसे उत्तरी प्रांत है । (स्पेन नहीं)

कैटेलोनिया स्पेन का एक स्वायत्त समुदाय है ।

अत: केवल दो जोड़े सही सुमेलित हैं।


69. वन्यजीव संरक्षण के बारे में भारतीय विधियों के सन्दर्भ में, निम्नलिखित कथनों पर विचार कीजिए:

  1. वन्यजीव, एकमात्र सरकार की संपत्ति हैं।
  2. जब किसी वन्यजीव को संरक्षित घोषित किया जाता है, तो यह जीव चाहे संरक्षित क्षेत्र में हो या उससे बाहर, समान संरक्षण का हकदार है ।
  3. किसी संरक्षित वन्यजीव के मानव जीवन के लिए ख़तरा बन जाने की आशका उस जीव को पकड़ने या मार दिए जाने का पर्याप्त आधार है ।

उपर्युक्त कथनों में कौन-सा/से सही है/हैं?

  1. 1 और 2
  2. केवल 2
  3. 1 और 3
  4. केवल 3

Sol. Ans.(a). एक महत्वपूर्ण फैसले में, बॉम्बे हाईकोर्ट ने फैसला सुनाया है कि बाघ सहित जंगली जानवरों को "सभी उद्देश्यों के लिए सरकारी संपत्ति" के रूप में माना जाना चाहिए। अतः, 1 सही है।

वन्य जीवन (संरक्षण) अधिनियम, 1972 - शिकार का प्रतिषेध। कोई भी व्यक्ति धारा 11 और धारा 12 के तहत प्रदान किए गए को छोड़कर, अनुसूची I, II, III और IV में निर्दिष्ट किसी भी जंगली जानवर का शिकार नहीं करेगा।

कुछ मामलों में जंगली जानवरों के शिकार की अनुमति -

  1. किसी अन्य कानून में कुछ समय के लिए लागू होने के बावजूद और अध्याय IV के प्रावधानों के अधीन -
  1. मुख्य वन्य जीवन वार्डन, यदि वह संतुष्ट है कि अनुसूची I में निर्दिष्ट कोई जंगली जानवर मानव जीवन के लिए खतरनाक हो गया है या इतना विकलांग या रोगग्रस्त है कि वसूली से परे है, लिखित आदेश और उसके कारणों को बताते हुए, किसी भी व्यक्ति को ऐसे जानवर का शिकार करने या ऐसे जानवर का शिकार करने की अनुमति देना: 1 [बशर्ते कि किसी भी जंगली जानवर को मारने का आदेश नहीं दिया जाएगा जब तक कि मुख्य वन्य जीवन वार्डन संतुष्ट न हो कि ऐसे जानवर को पकड़ा, शांत या स्थानांतरित नहीं किया जा सकता है: बशर्ते आगे कि ऐसे किसी भी पकड़े गए जानवर को तब तक कैद में नहीं रखा जाएगा जब तक कि मुख्य वन्य जीवन वार्डन संतुष्ट न हो कि ऐसे जानवर को जंगली में पुनर्वास नहीं किया जा सकता है और इसके कारणों को लिखित रूप में दर्ज किया गया है।

स्पष्टीकरण - खंड (a) के प्रयोजनों के लिए, ऐसे जानवर को पकड़ने या स्थानांतरित करने की प्रक्रिया, जैसा भी मामला हो, इस तरह से बनाई जाएगी कि उक्त जानवर को न्यूनतम आघात हो।]

अतः, 3 सही नहीं है।

स्रोत: https://zeenews.india.com/news/eco-news/treat-wild-animals-as-govt-property-for-all-purposes_769840.html


70. निम्नलिखित में से किस एक जीव की कुछ प्रजातियाँ कवकों के कृषकों के रूप में जानी जाती हैं?

  1. चींटी
  2. कॉक्रोच
  3. केकड़ा
  4. मकड़ी

Sol. Ans.(a). चींटियां दुनिया की सबसे अच्छी कवक किसान हैं। यह पारस्परिकता या सहजीवन का एक अद्भुत उदाहरण है।


71. निम्नलिखित कथनों पर विचार कीजिए:

  1. US फेडरल रिज़र्व की सख्त मुद्रा नीति पूँजी पलायन की ओर ले जा सकती है।
  2. पूँजी पलायन वर्तमान विदेशी वाणिज्यिक ऋणग्रहण (External Commercial Borrowings (ECBs) वाली फर्मों की ब्याज लागत को बढ़ा सकता है।
  3. घरेलू मुद्रा का अवमूल्यन, ECBs से संबद्ध मुद्रा जोखिम को घटाता है। उपर्युक्त कथनों में कौन-से सही हैं?
  1. केवल 1 और 2
  2. केवल 2 और 3
  3. केवल 1 और 3
  4. 1, 2 और 3

Sol. Ans.(a). कथन 1 और 2 सही हैं, लेकिन 3 नहीं है। यूएस फेडरल रिज़र्व की सख्त मौद्रिक नीति का मतलब होगा उच्च ब्याज दरें, जिसका अर्थ होगा भारत से बाहर पूंजी पलायन, और परिणामी प्रभाव।

घरेलू मुद्रा के मूल्यह्रास से घरेलू मुद्रा के संदर्भ में विदेशी मुद्रा ऋणों के मूल्य में वृद्धि होगी। इसलिए, स्थानीय मुद्रा का एक महत्वपूर्ण मूल्यह्रास बकाया ऋण के स्थानीय-मुद्रा मूल्य में वृद्धि में अनुवाद करेगा। नतीजतन, इससे घरेलू कर्जदारों की कर्ज चुकाने की क्षमता में गिरावट आएगी। ध्यान दें कि "बाहरी वाणिज्यिक उधार (ईसीबी)" विदेशी मुद्राओं में एक प्रकार का ऋण है, जो अनिवासी उधारदाताओं द्वारा किया जाता है।

( यह प्रश्न सीधे पीटी के पावरपैक कोर्स में शामिल था - यहां नामांकन करें -https://bit.ly/upscias )


72. निम्नलिखित राज्यों पर विचार कीजिए:

  1. आंध्र प्रदेश
  2. केरल
  3. हिमाचल प्रदेश
  4. त्रिपुरा

उपर्युक्त में से कितने आम तौर पर चाय-उत्पादक राज्य के रूप में जाने जाते हैं?

  1. केवल एक राज्य
  2. केवल दो राज्य
  3. केवल तीन राज्य
  4. सभी चारों राज्य

Sol. Ans.(a). असम, पश्चिम बंगाल, तमिलनाडु और केरल में भारत में कुल चाय उत्पादन का 98% हिस्सा है। अतः, सही उत्तर है (a)। [इस प्रश्न में "आम तौर पर" शब्द के कारण कुछ भ्रम है, इसलिए अन्य उत्तर भी संभव हैं]


73. निम्नलिखित कथनों पर विचार कीजिए:

  1. भारत में, साख़ क्षमता-निर्धारण एजेंसियाँ (क्रेडिट रेटिंग एजेंसीज़) भारतीय रिज़र्व बैंक द्वारा विनियमित होती हैं।
  2. ICRA नाम से जानी जाने वाली क्षमता-निर्धारण एजेंसी एक पब्लिक लिमिटेड कंपनी है ।
  3. ब्रिकवर्क रेटिंग्स एक भारतीय साख़ क्षमता-निर्धारण एजेंसी है।

उपर्युक्त कथनों में कौन-से सही हैं?

  1. केवल 1 और 2
  2. केवल 2 और 3
  3. केवल 1 और 3
  4. 1, 2 और 3

Sol. Ans.(b). भारत में, विभिन्न क्रेडिट रेटिंग एजेंसियों को सेबी (भारतीय प्रतिभूति और विनिमय बोर्ड) द्वारा विनियमित किया जाता है, न कि आरबीआई (भारतीय रिजर्व बैंक)। अतः, 1 सही नहीं है, और केवल संभव विकल्प (b) है, क्योंकि तीन विकल्पों को खारिज कर दिया गया है।

( यह प्रश्न सीधे पीटी के पावरपैक कोर्स में शामिल था - यहां नामांकन करें -https://bit.ly/upscias )


74. बैंक बोर्ड ब्यूरो (BBB)' के सन्दर्भ में, निम्नलिखित में कौन-से कथन सही हैं?

  1. RBI का गवर्नर BBB का चेयरमैन होता है ।
  2. BBB, सार्वजनिक क्षेत्रक बैंकों के अध्यक्षों के चयन के लिए संस्तुति करता है।
  3. BBB, सार्वजनिक क्षेत्रक बैंकों को कार्यनीतियों - और पूँजी-वर्धन योजनाओं को विकसित करने में मदद करता है।

नीचे दिए कूट का प्रयोग कर सही उत्तर चुनिए:

  1. केवल 1 और 2
  2. केवल 2 और 3
  3. केवल 1 और 3
  4. 1, 2 और 3

Sol. Ans.(b). आरबीआई के गवर्नर बैंक बोर्ड ब्यूरो (बीबीबी) के अध्यक्ष नहीं हैं। अतः, 1 सही नहीं है, और एकमात्र संभावित विकल्प (b) है।

( यह प्रश्न सीधे पीटी के पावरपैक कोर्स में शामिल था - यहां नामांकन करें -https://bit.ly/upscias )


75. परिवर्तनीय बॉन्ड के सन्दर्भ में, निम्नलिखित कथनों पर विचार कीजिए:

  1. चूँकि बॉन्ड को ईक्विटी के लिए बदलने का विकल्प है, परिवर्तनीय बॉन्ड अपेक्षाकृत कम ब्याज दर का भुगतान करते हैं।
  2. ईक्विटी के लिए बदलने का विकल्प बॉन्ड-धारक को बढ़ती हुई उपभोक्ता कीमतों से सहलग्नता (इंडेक्सेशन) की मात्रा प्रदान करता है ।

उपर्युक्त कथनों में कौन-सा/से सही है/हैं?

  1. केवल 1
  2. केवल 2
  3. 1 और 2 दोनों
  4. न तो 1, न ही 2

Sol. Ans.(c). परिवर्तनीय बांड बांड को सामान्य स्टॉक में बदलने के विकल्प के मूल्य के बदले में कम कूपन दर या वापसी की दर की पेशकश करते हैं। कंपनियों को लाभ होता है क्योंकि वे पारंपरिक बांड की पेशकश की तुलना में कम ब्याज दरों पर ऋण जारी कर सकते हैं लेकिन सभी कंपनियां परिवर्तनीय बांड की पेशकश नहीं करती हैं।

इक्विटी में बदलने का विकल्प बॉन्डधारक को बढ़ती उपभोक्ता कीमतों के लिए इंडेक्सेशन की एक डिग्री देता है। अतः, दोनों सही हैं।


76. निम्नलिखित पर विचार कीजिए:

  1. एशियाई अवसंरचना निवेश बैंक (एशियन इन्फ्रास्ट्रक्चर इन्वेस्टमेंट बैंक)
  2. प्रक्षेपास्त्र प्रौद्योगिकी नियंत्रण व्यवस्था (मिसाइल टेक्नोलॉजी कन्ट्रोल रिजीम)
  3. शंघाई सहयोग संगठन (शंघाई कोऑपरेशन ऑर्गेनाईज़ेशन) भारत उपर्युक्त में से किसका/किनका सदस्य है?

उपर्युक्त कथनों में कौन-सा/से सही है/हैं?

  1. केवल 1 और 2
  2. केवल 3
  3. केवल 2 और 3
  4. 1, 2 और 3

Sol. Ans.(d). भारत एशियन इन्फ्रास्ट्रक्चर इन्वेस्टमेंट बैंक (एआईआईबी) का सदस्य है और जब से बैंक ने अपना परिचालन शुरू किया है, तब से एआईआईबी ऋण देने वाला सबसे बड़ा उधारकर्ता रहा है।

एससीओ (शंघाई सहयोग संगठन) में वर्तमान में आठ सदस्य राज्य (चीन, भारत, कजाकिस्तान, किर्गिस्तान, रूस, पाकिस्तान, ताजिकिस्तान और उजबेकिस्तान) शामिल हैं, चार पर्यवेक्षक राज्य पूर्ण सदस्यता में शामिल होने में रुचि रखते हैं (अफगानिस्तान, बेलारूस, ईरान और मंगोलिया) और छह "संवाद भागीदार"। अतः 1 और 3 सही हैं। केवल संभव विकल्प (d) है।


77. निम्नलिखित कथनों पर विचार कीजिए:

  1. हाल के वर्षों में वियतनाम विश्व में सबसे तेजी से बढ़ती हुई अर्थव्यवस्थाओं में से एक रहा है ।
  2. वियतनाम का नेतृत्व बहु-दलीय राजनीतिक प्रणाली के द्वारा होता है।
  3. वियतनाम का आर्थिक विकास विश्वव्यापी पूर्ति श्रृंखलाओं के साथ इसके एकीकरण और निर्यात पर मुख्य ध्यान होने से जुड़ा है।
  4. लंबे समय से वियतनाम की निम्न श्रम लागतों और स्थिर विनिमय दरों ने वैश्विक निर्माताओं को आकर्षित किया है।
  5. हिंद-प्रशांत क्षेत्र का सर्वाधिक उत्पादक –सेवा सेक्टर वियतनाम में है।

उपर्युक्त कथनों में कौन-से सही हैं?

  1. 2 और 4
  2. 3 और 5
  3. 1, 3 और 4
  4. 1 और 2

Sol. Ans.(c). एक हालिया जीडीपी विकास सांख्यिकी रिपोर्ट में कहा गया है कि वियतनाम 2019 में 7.31% की वृद्धि दर के साथ सबसे तेजी से बढ़ती विश्व अर्थव्यवस्था है। इसने भारत और चीन सहित अन्य एशियाई अर्थव्यवस्थाओं की विकास दर को पीछे छोड़ दिया है, जो कि क्रमशः 4.5% और 6% सीएजीआर। अतः, 1 सही है।

वियतनाम एकात्मक, समाजवादी, साम्यवादी, एकदलीय राज्य है। अतः, 2 सही नहीं है। केवल संभव विकल्प (c) है।

स्रोत:
https://locus.sh/resources/bulletin/vietnams-economic-growth/#:~:text=A%20recent%20GDP%20growth%20statistics,%25%20and%206%25%20CAGR%20respectively


78. भारत में, निम्नलिखित में कौन मुद्रास्फीति को नियंत्रित कर कीमत स्थिरता बनाए रखने के लिए उत्तरदायी है?

  1. उपभोक्ता मामले विभाग
  2. व्यय प्रबंधन आयोग
  3. वित्तीय स्थिरता और विकास परिषद्
  4. भारतीय रिज़र्व बैंक

Sol. Ans.(d). आरबीआई और अन्य केंद्रीय बैंक की एक महत्वपूर्ण भूमिका मूल्य स्थिरता (कम और स्थिर मुद्रास्फीति) प्राप्त करने और आर्थिक उतार-चढ़ाव के प्रबंधन में मदद करने के लिए मौद्रिक नीति का संचालन करना है। यह एक आसान सवाल था।

( यह प्रश्न सीधे पीटी के पावरपैक कोर्स में शामिल था - यहां नामांकन करें -https://bit.ly/upscias )


79. नॉन-फंजिबल टोकेंस (Non-Fungible Tokens (NFTs)) के सन्दर्भ में, निम्नलिखित कथनों पर विचार कीजिए:-

  1. वे भौतिक परिसंपत्तियों के अंकीय निरूपण (डिजिटल रिप्रेजेंटेशन) को सुकर बनाते हैं ।
  2. वे अनन्य क्रिप्टोग्राफिक टोकेंस हैं जो किसी ब्लॉकचैन में विद्यमान हैं।
  3. उनका, तुल्यता पर, व्यापार या विनिमय किया जा सकता है और इसलिए उनका वाणिज्यिक लेन-देन के माध्यम के रूप में इस्तेमाल किया जा सकता हैं।

उपर्युक्त कथनों में कौन-से सही हैं?

  1. केवल 1 और 2
  2. केवल 2 और 3
  3. केवल 1 और 3
  4. 1, 2 और 3

Sol. Ans.(a). )एक नॉन-फनजीबल टोकन (एनएफटी) एक अद्वितीय डिजिटल संपत्ति है जो वास्तविक दुनिया की वस्तुओं जैसे कला, वीडियो क्लिप, संगीत, और बहुत कुछ के स्वामित्व का प्रतिनिधित्व करती है। एनएफटी उसी ब्लॉकचेन तकनीक का उपयोग करते हैं जो क्रिप्टोकरेंसी को शक्ति प्रदान करती है, लेकिन वे मुद्रा नहीं हैं। इसका उपयोग वाणिज्यिक लेनदेन के लिए एक माध्यम के रूप में नहीं किया जा सकता है।

( यह प्रश्न सीधे पीटी के पावरपैक कोर्स में शामिल था - यहां नामांकन करें -https://bit.ly/upscias )


80. निम्नलिखित युग्मों पर विचार कीजिए:

उपर्युक्त में से कितने युग्म सही सुमेलित नहीं हैं?

  1. केवल 1 और 2
  2. केवल 2 और 3
  3. केवल 1 और 3
  4. 1, 2 और 3

Sol. Ans.(c). घाटप्रभा- कर्नाटक, गांधी सागर- मध्य प्रदेश, इंदिरा सागर- मध्य प्रदेश, मैथोन- झारखंड। अतः तीन जोड़े सही नहीं हैं। यह एक कठिन प्रश्न है क्योंकि विकल्पों को समाप्त करने से इंकार किया जाता है। इसे ठीक से हल करने में सक्षम होने के लिए आपको सभी संबंधों को जानना चाहिए।


81. भारत सरकार अधिनियम 1919 में, प्रांतीय सरकार के कार्य “आरक्षित (रिज़र्ड)” और “अंतरित (ट्रांसफर्ड)" विषयों के अंतर्गत बाँटे गए थे । निम्नलिखित में कौन-से "आरक्षित" विषय माने गए थे?

  1. न्याय प्रशासन
  2. स्थानीय स्वशासन
  3. भू-राजस्व
  4. पुलिस

नीचे दिए कूट का प्रयोग कर सही उत्तर चुनिए:

  1. 1, 2 और 3
  2. 2, 3 और 4
  3. 1, 3 और 4
  4. 1, 2 और 4

Sol. Ans.(c). ' स्थानीय स्वशासन' को ' अंतरित सूची' में शामिल किया गया था। अतः, केवल संभावित विकल्प (c) है। इसे हल करने की एक छोटी सी तरकीब यह है कि बताए गए सभी विषयों पर गौर करें- न्याय प्रशासन, स्थानीय स्वशासन, भू-राजस्व और पुलिस। स्थानीय स्वशासन के अलावा, अन्य तीन स्पष्ट रूप से रणनीतिक प्रकृति के हैं जिन्हें ब्रिटिश स्थानीय राजनीतिक इकाइयों को सौंपना नहीं चाहेंगे।

( यह प्रश्न सीधे पीटी के पावरपैक कोर्स में शामिल था - यहां नामांकन करें -https://bit.ly/upscias )

स्रोत :
https://en.wikipedia.org/wiki/Government_of_India_Act_1919#:~:text=In%20each%20such%20province%2C%20control,The%20Provincial%20Councils%20were%20enlarged


82. मध्यकालीन भारत में, शब्द “फणम”किसे निर्दिष्ट करता था?

  1. पहनावा
  2. सिक्के
  3. आभूषण
  4. हथियार

Sol. Ans.(b). फणम मद्रास प्रेसीडेंसी जो अब तमिलनाडु, भारत का हिस्सा है और त्रावणकोर राज्य, जो अब केरल, भारत का हिस्सा है, में जारी एक मुद्रा थी। यह गंगा राजवंश काल में भी था।


https://en.wikipedia.org/wiki/Fanam

( यह प्रश्न सीधे पीटी के पावरपैक कोर्स में शामिल था - यहां नामांकन करें -https://bit.ly/upscias )


83. निम्नलिखित स्वतन्त्रता सेनानियों पर विचार कीजिए:

  1. बारीन्द्र कुमार घोष
  2. जोगेश चन्द्र चटर्जी
  3. रास बिहारी बोस

उपर्युक्त में से कौन ग़दर पार्टी के साथ सक्रिय रूप से जुड़ा था/जुड़े थे?

  1. 1 और 2
  2. केवल 2
  3. 1 और 3
  4. केवल 3

Sol. Ans.(d). रास बिहारी बोस (25 मई 1886 - 21 जनवरी 1945) ब्रिटिश राज के खिलाफ एक भारतीय क्रांतिकारी नेता थे। वह ग़दर विद्रोह के प्रमुख आयोजकों में से एक थे और उन्होंने विनायक दामोदर सावरकर की सैन्यीकरण नीति के आधार पर द्वितीय विश्व युद्ध के दौरान पहली भारतीय राष्ट्रीय सेना की स्थापना की। जोगेश चंद्र चटर्जी हिंदुस्तान रिपब्लिकन एसोसिएशन (HRA) से जुड़े थे।

( यह प्रश्न सीधे पीटी के पावरपैक कोर्स में शामिल था - यहां नामांकन करें -https://bit.ly/upscias )

स्रोत: https://en.wikipedia.org/wiki/Jogesh_Chandra_Chatterjee

स्रोत: https://en.wikipedia.org/wiki/Barindra_Kumar_Ghosh

स्रोत: https://en.wikipedia.org/wiki/Rash_Behari_Bose


84.क्रिप्स मिशन के प्रस्तावों के सन्दर्भ में, निम्नलिखित कथनों पर विचार कीजिए:

  1. संविधान सभा में प्रांतीय विधान-सभाओं और साथ ही भारतीय रियासतों द्वारा नामित सदस्य होंगे।
  2. नया संविधान स्वीकार करने के लिए जो भी प्रांत तैयार नहीं होगा, उसे यह अधिकार होगा. कि अपनी भावी स्थिति के बारे में ब्रिटेन के साथ अलग संधि पर हस्ताक्षर करे।

उपर्युक्त कथनों में कौन-सा/से सही है/हैं?

  1. केवल 1
  2. केवल 2
  3. 1 और 2 दोनों
  4. न तो 1, न ही 2

Sol. Ans.(c). क्रिप्स मिशन के प्रस्ताव थे -

  1. एक भारतीय डोमीनियन की स्थापना। इस डोमीनियन को ब्रिटिश राष्ट्रमंडल के साथ रहने या इससे अलग होने की स्वतंत्रता होगी। यह अंतरराष्ट्रीय संगठनों में भाग लेने के लिए भी स्वतंत्र होगा।
  2. देश के लिए एक नया संविधान बनाने के लिए एक संविधान सभा का गठन किया जाएगा। इस सभा में प्रांतीय विधानसभाओं द्वारा चुने गए सदस्य और राजकुमारों द्वारा मनोनीत भी होंगे ।
  3. कोई भी प्रांत जो भारतीय शासन में शामिल होने के लिए तैयार नहीं है, एक अलग संघ बना सकता है और उसका एक अलग संविधान हो सकता है।
  4. संविधान सभा और ब्रिटिश सरकार के बीच बातचीत से सत्ता के हस्तांतरण और अल्पसंख्यकों के अधिकारों की रक्षा की जाएगी।
  5. इस बीच, जब तक यह नया संविधान लागू नहीं हुआ, तब तक भारत की रक्षा पर अंग्रेजों का नियंत्रण रहेगा और गवर्नर-जनरल की शक्तियाँ अपरिवर्तित रहेंगी।
  6. कोई भी प्रांत जो भारतीय प्रभुत्व में शामिल होने के लिए तैयार नहीं है, एक अलग संघ बना सकता है और उसका एक अलग संविधान हो सकता है।
  7. अत: दोनों कथन सही हैं।

    ( यह प्रश्न सीधे पीटी के पावरपैक कोर्स में शामिल था - यहां नामांकन करें -https://bit.ly/upscias )


85. भारतीय इतिहास के सन्दर्भ में, निम्नलिखित मूलग्रंथों पर - विचार कीजिए:

  1. नेत्तिपकरण
  2. परिशिष्टपर्वन
  3. अवदानशतक
  4. त्रिशष्टिलक्षण महापुराण

उपर्युक्त में कौन-से जैन ग्रन्थ हैं?

  1. 1, 2 और 3
  2. केवल 2 और 4
  3. 1, 3 और 4
  4. 2, 3 और 4

Sol. Ans.(b). अवदानशतक या "सेंचुरी ऑफ नोबल डीड्स (अवदान)" संस्कृत में एक सौ बौद्ध किंवदंतियों का एक संकलन है, जो लगभग उसी समय अशोकवदन के रूप में है। अतः, 3 जैन पाठ नहीं है, और इसलिए तीन विकल्पों को खारिज कर दिया गया है। केवल संभव विकल्प (b) है । प्राचीन और मध्यकालीन भारत से एक अच्छा प्रश्न।

स्रोत: https://en.wikipedia.org/wiki/Avadanasataka


86. भारतीय इतिहास के सन्दर्भ में, निम्नलिखित युग्मों पर विचार कीजिए:

उपर्युक्त युग्मों में से कितने युग्म सही सुमेलित हैं?

  1. कोई भी युग्म नहीं
  2. केवल एक युग्म
  3. केवल दो युग्म
  4. सभी तीन युग्म

Sol. Ans.(c). आर्यदेव (तीसरी शताब्दी सीई) एक महायान बौद्ध भिक्षु, नागार्जुन के शिष्य और एक मध्यमा दार्शनिक थे। अधिकांश स्रोत इस बात से सहमत हैं कि वह "सिहला" से थे, जिसे कुछ विद्वान श्रीलंका के साथ पहचानते हैं।

दिग्नागा (480 ईस्वी - 540 ईस्वी) एक भारतीय बौद्ध विद्वान और भारतीय तर्कशास्त्र के बौद्ध संस्थापकों में से एक थे।

श्री रंगनाथमुनि, जिन्हें लोकप्रिय रूप से श्रीमन नाथमुनि (823 ईस्वी-951 ईस्वी) के नाम से जाना जाता है, एक वैष्णव धर्मशास्त्री थे जिन्होंने नलयिर दिव्य प्रबंधम को एकत्र और संकलित किया था।

केवल दो जोड़े सुमेलित हैं।


87. भारतीय इतिहास के सन्दर्भ में, निम्नलिखित कथनों पर विचार कीजिए:

  1. भारत पर पहला मंगोल आक्रमण जलालुद्दीन खिलज़ी के राज्यकाल में हुआ।
  2. अलाउद्दीन खिलज़ी के राज्य-काल में, एक मंगोल आक्रमण दिल्ली तक आ पहुँचा और उस शहर पर घेरा डाल दिया।
  3. मुहम्मद-बिन-तुगलक मंगोलों से अपने राज्य के कुछ उत्तरी-पश्चिमी भाग अस्थायी रूप से हार गया था।

उपर्युक्त कथनों में कौन-सा/से सही है/हैं?

  1. 1 और 2
  2. केवल 2
  3. 1 और 3
  4. केवल 3

Sol. Ans.(b and d). पहला (संभव) मंगोल आक्रमण इल्तुतमिश के शासनकाल में हुआ था, हालांकि चंगेज खान सिंधु से लौट गया था। लेकिन बाद में, जलाल-उद-दीन के शासनकाल से पहले मंगोलों द्वारा अन्य हमले किए गए थे। अला-उद-दीन खिलजी के शासनकाल के दौरान मंगोल हमले ने दिल्ली शहर को घेर लिया था। अतः, 2 सही है। कथन 3 भी सही है। अतः कोई भी विकल्प सही नहीं है।


88. भारतीय इतिहास के सन्दर्भ में, निम्नलिखित में से कौन “कुलाह-दारन”कहलाते थे?

  1. अरब व्यापारी
  2. कलंदर
  3. फारसी खुशनवीस
  4. सय्यद

Sol. Ans.(d). सैय्यद ने अपनी बेटी फातिमा के माध्यम से पैगंबर से अपने वंश का दावा किया। मुस्लिम समाज में उनका विशेष सम्मान था। यहां तक कि तैमूरलेन ने भी भारत में अपने आक्रमण के दौरान सैयदों के जीवन की रक्षा की, हालांकि उनकी नीति सामान्य वध की थी। सैय्यद एक नुकीली टोपी (कुलाह) पहनते थे और दिल्ली सल्तनत के दौरान उन्हें 'कुलाह दरन' के नाम से जाना जाता था।

स्रोत:
https://books.google.co.in/books?id=nMWSQuf4oSIC&printsec=frontcover&source=gbs_ge_summary_r&cad=0#v=onepage&q=Kulah%20Daran&f=false


88. भारतीय इतिहास के सन्दर्भ में, निम्नलिखित में से कौन “कुलाह-दारन”कहलाते थे?

  1. अरब व्यापारी
  2. कलंदर
  3. फारसी खुशनवीस
  4. सय्यद

Sol. Ans.(d). सैय्यद ने अपनी बेटी फातिमा के माध्यम से पैगंबर से अपने वंश का दावा किया। मुस्लिम समाज में उनका विशेष सम्मान था। यहां तक कि तैमूरलेन ने भी भारत में अपने आक्रमण के दौरान सैयदों के जीवन की रक्षा की, हालांकि उनकी नीति सामान्य वध की थी। सैय्यद एक नुकीली टोपी (कुलाह) पहनते थे और दिल्ली सल्तनत के दौरान उन्हें 'कुलाह दरन' के नाम से जाना जाता था।

स्रोत:
https://books.google.co.in/books?id=nMWSQuf4oSIC&printsec=frontcover&source=gbs_ge_summary_r&cad=0#v=onepage&q=Kulah%20Daran&f=false


89. भारतीय इतिहास के सन्दर्भ में, निम्नलिखित कथनों पर विचार कीजिए:

  1. डच लोगों ने पूर्वी तटीय क्षेत्रों में गजपति शासकों द्वारा प्रदान की गई जमीनों पर अपनी फैक्टरियाँ/गोदाम स्थापित किए ।
  2. अल्फोंसो दे अलबुकर्क ने बीजापुर सल्तनत से गोआ को छीन लिया था।
  3. अंग्रेज़ी ईस्ट इंडिया कंपनी ने मद्रास में विजयनगर साम्राज्य के एक प्रतिनिधि से पट्टे पर ली गई ... जमीन के एक प्लॉट पर फैक्टरी स्थापित की थी।

उपर्युक्त कथनों में कौन-से सही हैं?

  1. केवल 1 और 2
  2. केवल 2 और 3
  3. केवल 1 और 3
  4. 1, 2 और 3

Sol. Ans.(b). पहला डच उपनिवेश 1602 में स्थापित किया गया था जबकि गजपति साम्राज्य 1541 में समाप्त हुआ था। अंतिम शासक कखरुआ देव था। अतः 1 सही नहीं हो सकता। अतः तीन विकल्पों को तुरंत खारिज कर दिया जाता है।

अल्फांसो डी अल्बुकर्क ने विजयनगर साम्राज्य की मदद से बीजापुर सल्तनत के राजा आदिल शाही से गोवा पर कब्जा कर लिया। अतः, 2 सही है।

1639 में मद्रास में विजयनगर साम्राज्य के प्रतिनिधियों, जिसे नायक कहा जाता है, से लीज पर ली गई भूमि पर एक कारखाना स्थापित किया। अतः, 3 सही है।

Source : https://www.thehindu.com/features/kids/how-madras-came-to-be-376-years-ago/article7561549.ece


90. कौटिल्य अर्थशास्त्र के अनुसार, निम्नलिखित में कौन-से सही हैं?

  1. न्यायिक दंड के परिणामस्वरूप कोई व्यक्ति दास हो सकता था।
  2. स्त्री दास अपने मालिक के संसर्ग से पुत्र जनन पर कानूनी तौर पर मुक्त हो जाती थी।
  3. यदि स्त्री दास का मालिक उस स्त्री से पैदा हुए - पुत्र का पिता हो, तो उस पुत्र को मालिक का पुत्र होने का कानूनी हक़ मिलता था

उपर्युक्त कथनों में कौन-से सही हैं?

  1. केवल 1 और 2
  2. केवल 2 और 3
  3. केवल 1 और 3
  4. 1, 2 और 3

Sol. Ans.(d). अर्थशास्त्र कहता है कि एक आदमी या तो जन्म से, स्वेच्छा से खुद को बेचकर, युद्ध में कैद करके, या न्यायिक दंड के परिणामस्वरूप गुलाम हो सकता है। दासता एक मान्यता प्राप्त संस्था थी और स्वामी और दास के बीच कानूनी संबंध स्पष्ट रूप से परिभाषित थे जैसे यदि एक महिला दास ने अपने मालिक को एक बेटा पैदा किया, अतः न केवल वह कानूनी रूप से स्वतंत्र थी बल्कि बच्चा मालिक के बेटे की कानूनी स्थिति का हकदार था। अत: तीनों कथन सही हैं।

स्रोत: https://ncjindalps.com/pdf/HUMANITIES/The%20Kautilya%20Arthashastra%20-%20Chanakya.pdf

स्रोत: http://indiansaga.com/others/slavery.html


91. भारत में, निम्नलिखित में कौन एक, उन फैक्टरियों में जिनमें कामगार नियुक्त हैं, औद्योगिक विवादों, समापनों, छंटनी और कामबंदी के विषय में सूचनाओं को संकलित करता है?

  1. केन्द्रीय सांख्यिकी कार्यालय
  2. उद्योग संवर्धन और आंतरिक व्यापार विभाग
  3. श्रम ब्यूरो
  4. राष्ट्रीय तकनीकी जनशक्ति सूचना प्रणाली

Sol. Ans.(c). श्रम ब्यूरो औद्योगिक विवादों, बंद होने, छंटनी और छंटनी पर जानकारी संकलित करता है।

स्रोत: http://labourbureaunew.gov.in/UserContent/Statistics_ID_Layoffs_2011.pdf


92. भारत में, कोयला नियंत्रक संगठन (Coal Controller's Organization (CCO)) की क्या भूमिका है?

  1. cco भारत सरकार में कोयला सांख्यिकी का प्रमुख स्रोत है।
  2. यह बद्ध कोयला/लिग्नाइट खंड के विकास की प्रगति का मॉनीटरन करता है।
  3. यह कोयलायुक्त क्षेत्रों के अधिग्रहण के संबंध में सरकार की अधिसूचना के प्रति किसी आपत्ति का अनुश्रवण करता है।
  4. यह सुनिश्चित करता है कि कोयला खनन कंपनियाँ विहित समय में अंतिम उपभोक्ताओं को कोयला वितरण करें।

नीचे दिए कूट का प्रयोग कर सही उत्तर चुनिए:

  1. 1, 2 और 3
  2. केवल 3 और 4
  3. केवल 1 और 2
  4. 1, 2 और 4

Sol. Ans.(a). सांख्यिकी संग्रह अधिनियम, 2008 के तहत - कोयला और लिग्नाइट सांख्यिकी के संबंध में कोयला नियंत्रक को सांख्यिकीय प्राधिकरण बनाया गया है। वार्षिक कोयला और लिग्नाइट सर्वेक्षण करने और अनंतिम कोयला सांख्यिकी और भारत की कोयला निर्देशिका के प्रकाशन की जिम्मेदारी सौंपी। केंद्र और राज्य सरकार के विभिन्न मंत्रालयों, राष्ट्रीय और अंतर्राष्ट्रीय संगठनों को मासिक कोयला डेटा प्रस्तुत करना। कोल वाशरीज से संबंधित सांख्यिकी का संग्रह।

कोयला असर क्षेत्र (अधिग्रहण एवं विकास) अधिनियम, 1957 के तहत कोयला नियंत्रक इस अधिनियम के तहत सक्षम प्राधिकारी है जो कोयला आधारित भूमि के अधिग्रहण से संबंधित केंद्र सरकार की अधिसूचना पर किसी भी आपत्ति की सुनवाई करता है और अपनी रिपोर्ट केंद्र सरकार को प्रस्तुत करता है। कोकिंग कोल माइन्स (राष्ट्रीयकरण) अधिनियम, 1972, नॉन-कोकिंग कोल माइन्स (राष्ट्रीयकरण) अधिनियम, 1973 और कोयला खान (विशेष प्रावधान) अधिनियम, 2015 के तहत। कोयला नियंत्रक दावों के मामलों को निपटाने के लिए भुगतान आयुक्त के रूप में भी कार्य करता है। उपरोक्त अधिनियमों के तहत पूर्व-राष्ट्रीयकरण अवधि के कोलियरी मालिकों की।

स्रोत: http://www.coalcontroller.gov.in/pages/display/5-functionsresponsibilities


93. यदि किसी विशिष्ट क्षेत्र को भारत के संविधान की पाँचवीं अनुसूची के अधीन लाया जाए, तो निम्नलिखित कथनों में कौन-सा एक, इसके परिणाम को सर्वोत्तम रूप से प्रतिबिंबित करता है?

  1. इससे जनजातीय लोगों की जमीनें गैर-जनजातीय लोगों को अंतरित करने पर रोक लगेगी।
  2. इससे उस क्षेत्र में एक स्थानीय स्वशासी निकाय का सृजन होगा।
  3. इससे वह क्षेत्र संघ राज्यक्षेत्र में बदल जाएगा।
  4. जिस राज्य के पास ऐसे क्षेत्र होंगे, उसे विशेष कोटि का राज्य घोषित किया जाएगा।

Sol. Ans.(a). संविधान की पांचवीं अनुसूची अनुसूचित क्षेत्रों के साथ-साथ प्रशासन और नियंत्रण से संबंधित है। यह असम, मेघालय, त्रिपुरा और मिजोरम राज्यों के अलावा किसी भी राज्य में रहने वाली अनुसूचित जनजाति या अनुसूचित जनजातियों के सदस्यों द्वारा या उनके बीच भूमि के हस्तांतरण को प्रतिबंधित करता है।

( यह प्रश्न सीधे पीटी के पावरपैक कोर्स में शामिल था - यहां नामांकन करें -https://bit.ly/upscias )

स्रोत:
https://vikaspedia.in/social-welfare/scheduled-tribes-welfare/fifth-schedule-areas#:~:text=The%20Fifth%20Schedule%20of%20the%20Constitution%20deals%20with%20the%20administration,%2C%20Meghalaya%2C%20Tripura%20and%20Mizoram


94. निम्नलिखित कथनों पर विचार कीजिए:

  1. भारत स्वच्छता गठबंधन धारणीय स्वच्छता को संवर्धित करने वाला प्लेटफॉर्म है और भारत सरकार तथा विश्व स्वास्थ्य संगठन द्वारा इसका वित्तपोषण होता है।
  2. राष्ट्रीय नगर कार्य संस्थान भारत सरकार में आवासन एवं शहरी कार्य मंत्रालय का शीर्षस्थ निकाय है, और यह शहरी भारत की चुनौतियों का समाधान करने के नवप्रवर्तक हल उपलब्ध कराता है।

उपर्युक्त कथनों में कौन-सा/से सही है/हैं?

  1. केवल 1
  2. केवल 2
  3. 1 और 2 दोनों
  4. न तो 1, न ही 2

Sol. Ans.(b). फेडरेशन ऑफ इंडियन चैंबर कॉमर्स एंड इंडस्ट्री (फिक्की) में जून 2015 में लॉन्च किया गया भारत स्वच्छता गठबंधन (आईएससी), उत्प्रेरक क्रियाओं की एक रेंज के माध्यम से एक आम मंच पर कई संगठनों को लाकर सुरक्षित और टिकाऊ स्वच्छता को सक्षम और समर्थन करता है। इनमें निजी क्षेत्र पर ध्यान केंद्रित करते हुए WASH वित्तपोषण को अनलॉक करने का समर्थन करना, स्थायी स्वच्छता पर प्रवचन का नेतृत्व करने के लिए संबद्ध संगठनों के साथ साझेदारी करना शामिल है; स्वच्छता वकालत में सर्वोत्तम प्रथाओं का आयोजन, क्यूरेटिंग और प्रसार करना - अंतरिक्ष और संबद्ध मंचों पर भागीदारी के माध्यम से स्वच्छता के नीतिगत पहलुओं में इनपुट प्रदान करना। अतः यह किसी भी सरकार द्वारा वित्त पोषित नहीं है। भारत के या डब्ल्यूएचओ द्वारा। कथन 1 सही नहीं है। एनआईयूए को भारत सरकार की शहरी विकास योजनाओं में समर्थन और मार्गदर्शन करने के लिए एक शीर्ष निकाय के रूप में नियुक्त किया गया था। तब से, इसने आवास और शहरी मामलों के मंत्रालय के साथ मिलकर काम किया है। कथन 2 सही है। यह "सरकारी योजनाओं" का सवाल है।

स्रोत: http://urbanrivers.niua.org/node/46

स्रोत: https://www.indiasanitationcoalition.org/who-we-are.html


95. निम्नलिखित में कौन-सा एक, पर्यावरण (संरक्षण) अधिनियम, 1986 के अधीन गठित किया गया है?

  1. केन्द्रीय जल आयोग
  2. केन्द्रीय भूजल बोर्ड
  3. केन्द्रीय भूजल प्राधिकरण
  4. राष्ट्रीय जल विकास अभिकरण

Sol. Ans.(c). केंद्रीय भूजल प्राधिकरण (सीजीडब्ल्यूए) का गठन पर्यावरण (संरक्षण) अधिनियम, 1986 की धारा 3 की उप-धारा (3) के तहत भूजल विकास और प्रबंधन के विनियमन और नियंत्रण के उद्देश्य से किया गया था। देश।

( यह प्रश्न सीधे पीटी के पावरपैक कोर्स में शामिल था - यहां नामांकन करें -https://bit.ly/upscias )


96. “संयुक्त राष्ट्र प्रत्यय समिति (युनाईटेड नेशंस क्रेडेंशियल्स कमिटी)” के सन्दर्भ में, निम्नलिखित कथनों पर विचार कीजिए:

  1. यह संयुक्त राष्ट्र (UN) सुरक्षा परिषद् द्वारा स्थापित समिति है और इसके पर्यवेक्षण के अधीन काम करती है।
  2. पारंपरिक रूप से प्रति वर्ष मार्च, जून और सितंबर में इसकी बैठक होती है।
  3. यह महासभा को अनुमोदन हेतु रिपोर्ट प्रस्तुत करने से पूर्व सभी UN सदस्यों के प्रत्ययों का आकलन करती है।

उपर्युक्त कथनों में कौन-सा/से सही है/हैं?

  1. केवल 3
  2. 1 और 3
  3. 2 और 3
  4. 1 और 2

Sol. Ans.(a). संयुक्त राष्ट प्रत्यय समिति, यूएनजीए का हिस्सा है, न कि संयुक्त राष्ट सुरक्षा परिषद जैसा कि प्रश्न में दिया गया है। अतः, 1 सही नहीं है। नियमित अंतराल पर मिलने का कोई जिक्र नहीं है। अतः, 2 सही नहीं है।

स्रोत: https://www.un.org/en/ga/credentials/credentials.shtml


97. निम्नलिखित में से कौन-सा एक कथन 'ध्रुवीय कोड (Polar Code)' का सर्वोत्तम वर्णन करता है?

  1. ध्रुवीय जलराशियों में परिचालन कर रहे जहाज़ों के लिए यह सुरक्षा का अंतर्राष्ट्रीय कोड है ।
  2. यह उत्तरी ध्रुव के आसपास के देशों का ध्रुवीय क्षेत्र में अपने राज्यक्षेत्रों के सीमांकन का समझौता
  3. यह उत्तरी ध्रुव और दक्षिणी ध्रुव में अनुसंधान करने वाले वैज्ञानिकों के देशों द्वारा अपनाए जाने वाले मानकों का समुच्चय है।
  4. यह आर्कटिक कौंसिल के सदस्य देशों का व्यापारिक और सुरक्षा समझौता है ।

Sol. Ans.(a). ध्रुवीय जल (ध्रुवीय कोड) में संचालित जहाजों के लिए आईएमओ का अंतर्राष्ट्रीय कोड समुद्र में जीवन की सुरक्षा के लिए अंतर्राष्ट्रीय सम्मेलन (सोलास) और जहाजों से प्रदूषण की रोकथाम के लिए अंतर्राष्ट्रीय सम्मेलन (एमएआरपीओएल) दोनों के तहत अनिवार्य है। ) पोलर कोड में डिजाइन, निर्माण, उपकरण, परिचालन, प्रशिक्षण, खोज और बचाव और दो ध्रुवों के आस-पास के दुर्गम जल में चलने वाले जहाजों के लिए प्रासंगिक पर्यावरण संरक्षण मामलों की पूरी श्रृंखला शामिल है। ध्रुवीय जल (ध्रुवीय कोड) में संचालित जहाजों के लिए IMO का अंतर्राष्ट्रीय कोड समुद्र में जीवन की सुरक्षा के लिए अंतर्राष्ट्रीय सम्मेलन (SOLAS) और जहाजों से प्रदूषण की रोकथाम के लिए अंतर्राष्ट्रीय सम्मेलन (MARPOL) दोनों के तहत अनिवार्य है।

स्रोत: https://www.imo.org/en/OurWork/Safety/Pages/polar-code.aspx


98. संयुक्त राष्ट्र महासभा के सन्दर्भ में, निम्नलिखित कथनों पर विचार कीजिए:

  1. UN महासभा, गैर-सदस्य राज्यों को प्रेक्षक स्थिति प्रदान कर सकती है।
  2. अंत:सरकारी संगठन UN महासभा में प्रेक्षक स्थिति पाने का प्रयत्न कर सकते हैं ।
  3. UN महासभा में स्थायी प्रेक्षक UN मुख्यालय में मिशन बनाए रख सकते हैं।

उपर्युक्त कथनों में कौन-से सही हैं?

  1. केवल 1 और 2
  2. केवल 2 और 3
  3. केवल 1 और 3
  4. 1, 2 और 3

Sol. Ans.(d). संयुक्त राष्ट्र महासभा गैर-सदस्य राज्यों , अंतर्राष्ट्रीय संगठनों और अन्य संस्थाओं को स्थायी पर्यवेक्षक का दर्जा दे सकती है। स्थायी पर्यवेक्षक महासभा के सत्रों और कामकाज में भाग ले सकते हैं और संयुक्त राष्ट्र मुख्यालय में मिशन बनाए रख सकते हैं ।

स्रोत: https://ask.un.org/faq/14519


99. भारत में “चाय बोर्ड” के सन्दर्भ में, निम्नलिखित कथनों पर विचार कीजिए:

  1. चाय बोर्ड सांविधिक निकाय है ।
  2. यह कृषि एवं किसान कल्याण मंत्रालय से संलग्न ___नियामक निकाय है।
  3. चाय बोर्ड का प्रधान कार्यालय बेंगलुरु में स्थित
  4. इस बोर्ड के दुबई और मॉस्को में विदेशी कार्यालय

उपर्युक्त कथनों में कौन-से सही हैं?

  1. 1 और 3
  2. 2 और 4
  3. 3 और 4
  4. 1 और 4

Sol. Ans.(d). चाय अधिनियम 1953 की धारा 4 के तहत स्थापित चाय बोर्ड का गठन 1 अप्रैल 1954 को किया गया था।

चाय बोर्ड वाणिज्य मंत्रालय के तहत केंद्र सरकार के एक वैधानिक निकाय के रूप में कार्य कर रहा है।

चाय बोर्ड का प्रधान कार्यालय कोलकाता में स्थित है।

वर्तमान में टी बोर्ड के दुबई और मॉस्को में दो विदेशी कार्यालय हैं।

स्रोत: https://www.teaboard.gov.in/TEABOARDCSM/NA==


100. निम्नलिखित में कौन-सा एक, “ग्रीनवाशिंग”शब्द का सर्वोत्तम वर्णन है?

  1. मिथ्या रूप से यह प्रभाव व्यक्त करना कि कंपनी के उत्पाद पारिस्थितिक-अनुकूली (ईको-फ्रेंडली) और पर्यावरणीय रूप से उपयुक्त हैं
  2. किसी देश के वार्षिक वित्तीय विवरणों में पारिस्थितिक/पर्यावरणीय लागतों को शामिल नहीं करना
  3. आधारिक संरचना विकसित करते समय • अनर्थकारी पारिस्थितिक दुष्परिणामों की उपेक्षा करना
  4. किसी सरकारी परियोजना/कार्यक्रम में पर्यावरणीय लागतों के लिए अनिवार्य उपबंध करना

Sol. Ans.(a). ग्रीनवाशिंग एक गलत धारणा देने या कंपनी के उत्पादों के बारे में भ्रामक जानकारी प्रदान करने की प्रक्रिया है कि कैसे पर्यावरण की दृष्टि से बेहतर है। उपभोक्ताओं को यह विश्वास दिलाने के लिए कि कंपनी के उत्पाद पर्यावरण के अनुकूल हैं, एक निराधार दावा माना जाता है।

( यह प्रश्न सीधे पीटी के पावरपैक कोर्स में शामिल था - यहां नामांकन करें -https://bit.ly/upscias )

स्रोत: https://www.financialexpress.com/opinion/greenwashing-is-an-elusive-csr-attempt/1332364/



Exam Analysis HOME       2022 - Paper I - English       Paper II - E       Paper I - Hindi       Paper II - H      


Questions and Detailed Solutions are being continuously updated ... refresh and check. Comment and let us know your experience, answers and solutions too!